You are on page 1of 38

QUALIFICATIONS, Election, Term, Oath the Supreme Court as the PET, to undertake the Herculean task of

Note: Executive power is vested in the President (alone)- cabinet deciding election protests involving presidential and
members are alter-ego of the President but they are not the Chief vice-presidential candidates in accordance with the process outlined
Executive, serve at the pleasure of the President by former Chief Justice Roberto Concepcion. It was made in
response to the concern aired by delegate Jose E. Suarez that the
MACALINTAL V. PRESIDENTIAL ELECTORAL TRIBUNAL additional duty may prove too burdensome for the Supreme Court.
FACTS: This explicit grant of independence and of the plenary powers
A Motion for Reconsideration was filed by petitioner Atty. Romulo B. needed to discharge this burden justifies the budget allocation of
Macalintal from a decision dismissing his petition and declaring the the PET.
establishment of respondent Presidential Electoral Tribunal (PET) as
constitutional. The conferment of additional jurisdiction to the Supreme Court,
with the duty characterized as an "awesome" task, includes the
Petitioner reiterates his arguments on the alleged unconstitutional means necessary to carry it into effect under the doctrine of
creation of the PET that Section 4, Article VII of the Constitution necessary implication. We cannot overemphasize that the
does not provide for the creation of the PET. Thus, PET violates abstraction of the PET from the explicit grant of power to the
Section 12, Article VIII of the Constitution. Supreme Court, given our abundant experience, is not unwarranted.

To bolster his arguments that the PET is an illegal and unauthorized A plain reading of Article VII, Section 4, paragraph 7, readily reveals
progeny of Section 4, Article VII of the Constitution, petitioner a grant of authority to the Supreme Court sitting en banc. In the
invokes our ruling on the constitutionality of the Philippine Truth same vein, although the method by which the Supreme Court
Commission (PTC). Petitioner cites the concurring opinion of Justice exercises this authority is not specified in the provision, the grant of
Teresita J. Leonardo-de Castro that the PTC is a public office which power does not contain any limitation on the Supreme Court's
cannot be created by the President, the power to do so being exercise thereof. The Supreme Court's method of deciding
lodged exclusively with Congress. Thus, petitioner submits that if presidential and vice-presidential election contests, through the PET,
the President, as head of the Executive Department, cannot create is actually a derivative of the exercise of the prerogative conferred
the PTC, the Supreme Court, likewise, cannot create the PET in the by the aforequoted constitutional provision. Thus, the subsequent
absence of an act of legislature. directive in the provision for the Supreme Court to "promulgate its
rules for the purpose."
ISSUE:
Whether the establishment of respondent Presidential Electoral The conferment of full authority to the Supreme Court, as a PET, is
Tribunal (PET) is constitutional. equivalent to the full authority conferred upon the electoral
tribunals of the Senate and the House of Representatives, i.e., the
HELD: Senate Electoral Tribunal (SET) and the House of Representatives
The decision of the Court still stands on its constitutionality Electoral Tribunal (HRET), which we have affirmed on numerous
occasions.
POLITICAL LAW judicial power
It is also beyond cavil that when the Supreme Court, as PET, resolves
We reiterate that the PET is authorized by the last paragraph of a presidential or vice-presidential election contest, it performs what
Section 4, Article VII of the Constitution and as supported by the is essentially a judicial power.
discussions of the Members of the Constitutional Commission,
which drafted the present Constitution. With the explicit provision, the present Constitution has allocated to
the Supreme Court, in conjunction with latter's exercise of judicial
The explicit reference by the framers of our Constitution to power inherent in all courts, the task of deciding presidential and
constitutionalizing what was merely statutory before is not diluted vice-presidential election contests, with full authority in the exercise
by the absence of a phrase, line or word, mandating the Supreme thereof. The power wielded by PET is a derivative of the plenary
Court to create a Presidential Electoral Tribunal. judicial power allocated to courts of law, expressly provided in the
Constitution. On the whole, the Constitution draws a thin, but,
Suffice it to state that the Constitution, verbose as it already is, nevertheless, distinct line between the PET and the Supreme Court.
cannot contain the specific wording required by petitioner in order
for him to accept the constitutionality of the PET. We have previously declared that the PET is not simply an agency to
which Members of the Court were designated. Once again, the PET,
Judicial power granted to the Supreme Court by the same as intended by the framers of the Constitution, is to be an
Constitution is plenary. And under the doctrine of necessary institution independent, but not separate, from the judicial
implication, the additional jurisdiction bestowed by the last department, i.e., the Supreme Court. McCulloch v. State of
paragraph of Section 4, Article VII of the Constitution to decide Maryland proclaimed that "[a] power without the means to use it, is
presidential and vice-presidential elections contests includes the a nullity."
means necessary to carry it into effect. Thus:
The decision therein held that the PTC "finds justification under
Obvious from the foregoing is the intent to bestow independence to Section 17, Article VII of the Constitution." A plain reading of the
1
constitutional provisions, i.e., last paragraph of Section 4 and
Section 17, both of Article VII on the Executive Branch, reveals that Second Issue:
the two are differently worded and deal with separate powers of It is also beyond cavil that when the Supreme Court, as PET, resolves
the Executive and the Judicial Branches of government. And as a presidential or vice-presidential election contest, it performs what
previously adverted to, the basis for the constitution of the PET was, is essentially a judicial power. In the landmark case ofAngara v.
in fact, mentioned in the deliberations of the Members of the Electoral Commission,Justice Jose P. Laurel enucleated that "it
Constitutional Commission during the drafting of the present would be inconceivable if the Constitution had not provided for a
Constitution. mechanism by which to direct the course of government along
constitutional channels." In fact,Angarapointed out that "[t]he
The Motion for Reconsideration is denied. Constitution is a definition of the powers of government." And yet,
at that time, the 1935 Constitution did not contain the expanded
ISSUES: definition of judicial power found in Article VIII, Section 1,
Whether the creation of the Presidential Electoral Tribunal is paragraph 2 of the present Constitution.
unconstitutional for being a violation of paragraph 7, Section 4 of
Article VII of the 1987 Constitution
Privilege and Salary
Whether the designation of members of the supreme court as
members of the presidential electoral tribunal is unconstitutional Executive Immunity applies only while sitting as President
for being a violation of Section 12, Article VIII of the 1987 (only the President can invoke)
Constitution SOLIVEN V. MAKASIAR
167 SCRA 393 Political Law Constitutional Law Presidents
HELD: Immunity From Suit Must Be Invoked by the President
Constitutional Law Luis Beltran is among the petitioners in this case. He, together with
First Issue: others, was charged with libel by the then president Corzaon
Petitioner, a prominent election lawyer who has filed several cases Aquino. Cory herself filed a complaint-affidavit against him and
before this Court involving constitutional and election law issues, others. Makasiar averred that Cory cannot file a complaint affidavit
including, among others, the constitutionality of certain provisions because this would defeat her immunity from suit. He grounded his
of Republic Act (R.A.) No. 9189 (The Overseas Absentee Voting Act contention on the principle that a president cannot be sued.
of 2003),cannot claim ignorance of: (1) the invocation of our However, if a president would sue then the president would allow
jurisdiction under Section 4, Article VII of the Constitution; and (2) herself to be placed under the courts jurisdiction and conversely
the unanimous holding thereon. Unquestionably, theoverarching she would be consenting to be sued back. Also, considering the
frameworkaffirmed inTecson v. Commission on Electionsis that the functions of a president, the president may not be able to appear in
Supreme Court has original jurisdiction to decide presidential and court to be a witness for herself thus she may be liable for
vice-presidential election protests while concurrentlyacting as an contempt.
independent Electoral Tribunal.
ISSUE:
Verba legisdictates that wherever possible, the words used in the Whether or not such immunity can be invoked by Beltran, a person
Constitution must be given their ordinary meaning except where other than the president.
technical terms are employed, in which case the significance thus
attached to them prevails. However, where there is ambiguity or HELD:
doubt, the words of the Constitution should be interpreted in No. The rationale for the grant to the President of the privilege of
accordance with the intent of its framers orratio legis et anima. A immunity from suit is to assure the exercise of Presidential duties
doubtful provision must be examined in light of the history of the and functions free from any hindrance or distraction, considering
times, and the condition and circumstances surrounding the that being the Chief Executive of the Government is a job that, aside
framing of the Constitution. Last,ut magis valeat quam pereat the from requiring all of the office-holders time, also demands
Constitution is to be interpreted as a whole. undivided attention.
But this privilege of immunity from suit, pertains to the President by
By the same token, the PET is not a separate and distinct entity from virtue of the office and may be invoked only by the holder of the
the Supreme Court, albeit it has functions peculiar only to the office; not by any other person in the Presidents behalf. Thus, an
Tribunal. It is obvious that the PET was constituted in accused like Beltran et al, in a criminal case in which the President is
implementation of Section 4, Article VII of the Constitution, and it the complainant cannot raise the presidential privilege as a defense
faithfully complies not unlawfully defies the constitutional directive. to prevent the case from proceeding against such accused.
The adoption of a separate seal, as well as the change in the Moreover, there is nothing in our laws that would prevent the
nomenclature of the Chief Justice and the Associate Justices into President from waiving the privilege. Thus, if so minded the
Chairman and Members of the Tribunal, respectively, was designed President may shed the protection afforded by the privilege and
simply to highlight the singularity and exclusivity of the Tribunals submit to the courts jurisdiction. The choice of whether to exercise
functions as a special electoral court. the PET, as intended by the the privilege or to waive it is solely the Presidents prerogative. It is
framers of the Constitution, is to be an institutionindependent,but a decision that cannot be assumed and imposed by any other
not separate, from the judicial department,i.e., the Supreme Court. person.
2
extra constitutional and the legitimacy of the new government that
resulted from it cannot be the subject of judicial review
Estrada vs Desierto G.R. No. 146710-15; Estrada vs Arroyo intra constitutional and the resignation of the sitting President that
G.R. No. 146738, March 2 2001 it caused and the succession of the Vice President as President are
[Immunity from Suit; Resignation of the President; Justiciable subject to judicial review.
controversy] presented a political question; involves legal questions.
FACTS: The cases at bar pose legal and not political questions. The principal
It began in October 2000 when allegations of wrong doings issues for resolution require the proper interpretation of certain
involving bribe-taking, illegal gambling, and other forms of provisions in the 1987 Constitution: Sec 1 of Art II, and Sec 8 of Art
corruption were made against Estrada before the Senate Blue VII, and the allocation of governmental powers under Sec 11 of Art
Ribbon Committee. On November 13, 2000, Estrada was impeached VII. The issues likewise call for a ruling on the scope of presidential
by the Hor and, on December 7, impeachment proceedings were immunity from suit. They also involve the correct calibration of the
begun in the Senate during which more serious allegations of graft right of petitioner against prejudicial publicity.
and corruption against Estrada were made and were only stopped
on January 16, 2001 when 11 senators, sympathetic to the 2. Elements of valid resignation: (a)an intent to resign and (b) acts of
President, succeeded in suppressing damaging evidence against relinquishment. Both were present when President Estrada left the
Estrada. As a result, the impeachment trial was thrown into an Palace.
uproar as the entire prosecution panel walked out and Senate Totality of prior contemporaneous posterior facts and circumstantial
President Pimentel resigned after casting his vote against Estrada. evidence bearing material relevant issuesPresident Estrada is
deemed to have resigned constructive resignation.
On January 19, PNP and the AFP also withdrew their support for SC declared that the resignation of President Estrada could not be
Estrada and joined the crowd at EDSA Shrine. Estrada called for a doubted as confirmed by his leaving Malacaan Palace. In the press
snap presidential election to be held concurrently with release containing his final statement:
congressional and local elections on May 14, 2001. He added that 1. He acknowledged the oath-taking of the respondent as President;
he will not run in this election. On January 20, SC declared that the 2. He emphasized he was leaving the Palace for the sake of peace
seat of presidency was vacant, saying that Estrada constructively and in order to begin the healing process (he did not say that he
resigned his post. At noon, Arroyo took her oath of office in the was leaving due to any kind of disability and that he was going to
presence of the crowd at EDSA as the 14th President. Estrada and reassume the Presidency as soon as the disability disappears);
his family later left Malacaang Palace. Erap, after his fall, filed 3. He expressed his gratitude to the people for the opportunity to
petition for prohibition with prayer for WPI. It sought to enjoin the serve them as President (without doubt referring to the past
respondent Ombudsman from conducting any further proceedings opportunity);
in cases filed against him not until his term as president ends. He 4. He assured that he will not shirk from any future challenge that
also prayed for judgment confirming Estrada to be the lawful and may come in the same service of the country;
incumbent President of the Republic of the Philippines temporarily 5. He called on his supporters to join him in promotion of a
unable to discharge the duties of his office. constructive national spirit of reconciliation and solidarity.
Intent to resignmust be accompanied by act of
ISSUE(S): relinquishmentact or omission before, during and after January
1. WoN the petition presents a justiciable controversy. 20, 2001.
2. WoN Estrada resigned as President.
3. WoN Arroyo is only an acting President. 3. The Congress passed House Resolution No. 176 expressly stating
4. WoN the President enjoys immunity from suit. its support to Gloria Macapagal-Arroyo as President of the Republic
5. WoN the prosecution of Estrada should be enjoined due to of the Philippines and subsequently passed H.R. 178 confirms the
prejudicial publicity. nomination of Teofisto T. Guingona Jr. As Vice President. Senate
passed HR No. 83 declaring the Impeachment Courts as Functius
RULING: Officio and has been terminated. It is clear is that both houses of
1. Political questions- "to those questions which, under the Congress recognized Arroyo as the President. Implicitly clear in that
Constitution, are to be decided by the people in their sovereign recognition is the premise that the inability of Estrada is no longer
capacity, or in regard to which full discretionary authority has been temporary as the Congress has clearly rejected his claim of inability.
delegated to the legislative or executive branch of the government. The Court therefore cannot exercise its judicial power for this is
It is concerned with issues dependent upon the wisdom, not legality political in nature and addressed solely to Congress by
of a particular measure." constitutional fiat. In fine, even if Estrada can prove that he did
not resign, still, he cannot successfully claim that he is a President
Legal distinction between EDSA People Power I EDSA People Power on leave on the ground that he is merely unable to govern
II: temporarily. That claim has been laid to rest by Congress and the
EDSA I EDSA II decision that Arroyo is the de jure, president made by a co-equal
exercise of the people power of revolution which overthrew the branch of government cannot be reviewed by this Court.
whole government. exercise of people power of freedom of speech
and freedom of assemblyto petition the government for redress of 4. The cases filed against Estrada are criminal in character. They
grievances which only affected the office of the President. involve plunder, bribery and graft and corruption. By no stretch of
3
the imagination can these crimes, especially plunder which carries evaluation of intelligence reports and information regarding "illegal
the death penalty, be covered by the alleged mantle of immunity of activities affecting the national economy, such as, but not limited to,
a non-sitting president. He cannot cite any decision of this Court economic sabotage, smuggling, tax evasion, dollar salting."
licensing the President to commit criminal acts and wrapping him Consequently while in cases which involve state secrets it may be
with post-tenure immunity from liability. The rule is that unlawful sufficient to determine the circumstances of the case that there is
acts of public officials are not acts of the State and the officer who reasonable danger that compulsion of the evidence will expose
acts illegally is not acting as such but stands in the same footing as military matters without compelling production, no similar excuse
any trespasser. can be made for privilege resting on other considerations.

5. No. Case law will tell us that a right to a fair trial and the free
press are incompatible. Also, since our justice system does not use SENATE OF THE PHILIPPINES V. ERMITA
the jury system, the judge, who is a learned and legally enlightened 495 SCRA 170 Political Law Constitutional Law
individual, cannot be easily manipulated by mere publicity. The In 2005, scandals involving anomalous transactions about the North
Court also said that Estrada did not present enough evidence to Rail Project as well as the Garci tapes surfaced. This prompted the
show that the publicity given the trial has influenced the judge so as Senate to conduct a public hearing to investigate the said anomalies
to render the judge unable to perform. Finally, the Court said that particularly the alleged overpricing in the NRP. The investigating
the cases against Estrada were still undergoing preliminary Senate committee issued invitations to certain department heads
investigation, so the publicity of the case would really have no and military officials to speak before the committee as resource
permanent effect on the judge and that the prosecutor should be persons. Ermita submitted that he and some of the department
more concerned with justice and less with prosecution. heads cannot attend the said hearing due to pressing matters that
need immediate attention. AFP Chief of Staff Senga likewise sent a
similar letter. Drilon, the senate president, excepted the said
Executive Privilege requests for they were sent belatedly and arrangements were
already made and scheduled. Subsequently, GMA issued EO 464
ALMONTE VS VASQUEZ which took effect immediately.
G.R. No. 93567, May 23 1995 EO 464 basically prohibited Department heads, Senior officials of
Petitioners: Nerio Rogado, Chief Accountant; Elisa Rivera, Chief of executive departments who in the judgment of the department
the Records; Jose T. Almonte, EIIB Commissioner; Villamor Perez, heads are covered by the executive privilege; Generals and flag
Budget and Fiscal Management Division Chief; Respondent: officers of the Armed Forces of the Philippines and such other
Honorable Conrado M. Vasquez officers who in the judgment of the Chief of Staff are covered by the
executive privilege; Philippine National Police (PNP) officers with
FACTS: rank of chief superintendent or higher and such other officers who
Ombudsman Vasquez required Rogado and Rivera of Economic in the judgment of the Chief of the PNP are covered by the
Intelligence and Investigation Bureau (EIIB) to produce all executive privilege; Senior national security officials who in the
documents relating to Personal Service Funds yr. 1988 and all judgment of the National Security Adviser are covered by the
evidence for the whole plantilla of EIIB for 1988. The subpoena executive privilege; and Such other officers as may be determined
duces tecum was issued in connection with the investigation of by the President, from appearing in such hearings conducted by
funds representing savings from unfilled positions in the EIIB which Congress without first securing the presidents approval.
were legally disbursed. Almonte and Perez denied the anomalous The department heads and the military officers who were invited by
activities that circulate around the EIIB office. They moved to the Senate committee then invoked EO 464 to except themselves.
quash the subpoena duces tecum. They claim privilege of an agency Despite EO 464, the scheduled hearing proceeded with only 2
of the Government. military personnel attending. For defying President Arroyos order
barring military personnel from testifying before legislative inquiries
ISSUE: without her approval, Brig. Gen. Gudani and Col. Balutan were
Whether or not an Ombudsman can oblige the petitioners by virtue relieved from their military posts and were made to face court
of subpoena duces tecum to provide documents relating to martial proceedings. EO 464s constitutionality was assailed for it is
personal service and salary vouchers of EIIB employers. alleged that it infringes on the rights and duties of Congress to
conduct investigation in aid of legislation and conduct oversight
RULING: functions in the implementation of laws.
Yes. A government privilege against disclosure is recognized with
respect to state secrets bearing on military, diplomatic and similar ISSUE:
matters. This privilege is based upon public interest of such Whether or not EO 464 is constitutional.
paramount importance as in and of itself transcending the
individual interests of a private citizen, even though, as a HELD:
consequence thereof, the plaintiff cannot enforce his legal rights. The SC ruled that EO 464 is constitutional in part. To determine the
validity of the provisions of EO 464, the SC sought to distinguish
In the case at bar, there is no claim that military or diplomatic Section 21 from Section 22 of Art 6 of the 1987 Constitution. The
secrets will be disclosed by the production of records pertaining to Congress power of inquiry is expressly recognized in Section 21 of
the personnel of the EIIB. EIIB's function is the gathering and Article VI of the Constitution. Although there is no provision in the
4
Constitution expressly investing either House of Congress with such information is not as imperative as that of the President to
power to make investigations and exact testimony to the end that it whom, as Chief Executive, such department heads must give a
may exercise its legislative functions advisedly and effectively, such report of their performance as a matter of duty. In such instances,
power is so far incidental to the legislative function as to be implied. Section 22, in keeping with the separation of powers, states that
In other words, the power of inquiry with process to enforce it is Congress may only request their appearance. Nonetheless, when
an essential and appropriate auxiliary to the legislative function. A the inquiry in which Congress requires their appearance is in aid of
legislative body cannot legislate wisely or effectively in the absence legislation under Section 21, the appearance is mandatory for the
of information respecting the conditions which the legislation is same reasons stated inArnault.
intended to affect or change; and where the legislative body does
not itself possess the requisite information which is not
infrequently true recourse must be had to others who do possess Prohibitions (sec. 13; also part on chapter of civil service
it. commission)
Section 22 on the other hand provides for the Question Hour. The
Question Hour is closely related with the legislative power, and it is G. R. No. 85468, September 07, 1989
precisely as a complement to or a supplement of the Legislative DOROMA VS. SANDIGANBAYAN,Ombudsman and Special
Inquiry. The appearance of the members of Cabinet would be very, Prosecutor
very essential not only in the application of check and balance but FACTS:
also, in effect, in aid of legislation. Section 22 refers only to Quintin S. Doromal, a former Commissioner of the Presidential
Question Hour, whereas, Section 21 would refer specifically to Commission on Good Government (PCGG), for violation of the
inquiries in aid of legislation, under which anybody for that matter, Anti-Graft and Corrupt Practices Act (RA 3019), Sec. 3(h), in
may be summoned and if he refuses, he can be held in contempt of connection with his shareholdings and position as president and
the House. A distinction was thus made between inquiries in aid of director of the Doromal International Trading Corporation (DITC)
legislation and the question hour. While attendance was meant to which submitted bids to supply P61 million worth of electronic,
be discretionary in the question hour, it was compulsory in inquiries electrical, automotive, mechanical and airconditioning equipment
in aid of legislation. Sections 21 and 22, therefore, while closely to the Department of Education, Culture and Sports (or DECS) and
related and complementary to each other, should not be considered the National Manpower and Youth Council (or NMYC).
as pertaining to the same power of Congress. One specifically An information was then filed by the Tanodbayan against
relates to the power to conduct inquiries in aid of legislation, the Doromal for the said violation and a preliminary investigation was
aim of which is to elicit information that may be used for legislation, conducted.
while the other pertains to the power to conduct a question hour, The petitioner then filed a petition for certiorari and prohibition
the objective of which is to obtain information in pursuit of questioning the jurisdiction of the Tanodbayan to file the
Congress oversight function. Ultimately, the power of Congress to information without the approval of the Ombudsman.
compel the appearance of executive officials under Section 21 and The Supreme Court held that the incumbent Tanodbayan (called
the lack of it under Section 22 find their basis in the principle of Special Prosecutor under the 1987 Constitution and who is
separation of powers. supposed to retain powers and duties NOT GIVEN to the
While the executive branch is a co-equal branch of the legislature, it Ombudsman) is clearly without authority to conduct preliminary
cannot frustrate the power of Congress to legislate by refusing to investigations and to direct the filing of criminal cases with the
comply with its demands for information. When Congress Sandiganbayan, except upon orders of the Ombudsman.
exercises its power of inquiry, the only way for department heads to Subsequently annulling the information filed by the Tanodbayan.
exempt themselves therefrom is by a valid claim of privilege. They A new information, duly approved by the Ombudsman, was filed in
are not exempt by the mere fact that they are department heads. the Sandiganbayan, alleging that the Doromal, a public officer, being
Only one executive official may be exempted from this power the then a Commissioner of the Presidential Commission on Good
President on whom executive power is vested, hence, beyond the Government, did then and there wilfully and unlawfully, participate
reach of Congress except through the power of impeachment. It in a business through the Doromal International Trading
is based on her being the highest official of the executive branch, Corporation, a family corporation of which he is the President, and
and the due respect accorded to a co-equal branch of government which company participated in the biddings conducted by the
which is sanctioned by a long-standing custom. The requirement Department of Education, Culture and Sports and the National
then to secure presidential consent under Section 1, limited as it is Manpower & Youth Council, which act or participation is prohibited
only to appearances in the question hour, is valid on its face. For by law and the constitution.
under Section 22, Article VI of the Constitution, the appearance of The petitioner filed a motion to quash the information on the
department heads in the question hour is discretionary on their part. ground that it was invalid since there had been no preliminary
Section 1 cannot, however, be applied to appearances of investigation for the new information that was filed against him.
department heads in inquiries in aid of legislation. Congress is not The motion was denied by Sandiganbayan claiming that another
bound in such instances to respect the refusal of the department preliminary investigation is unnecessary because both old and new
head to appear in such inquiry, unless a valid claim of privilege is informations involve the same subject matter.
subsequently made, either by the President herself or by the
Executive Secretary. ISSUES:
When Congress merely seeks to be informed on how department 1. Whether or not the act of Doromal would constitute a violation of
heads are implementing the statutes which it has issued, its right to the Constitution.
5
2. Whether or not preliminary investigation is necessary even if both practice any other profession, participate in any business, or be
informations involve the same subject matter. financially interested in any contract with, or in any franchise, or
3. Whether or not the information shall be effected as invalid due to special privilege granted by the Government or any subdivision,
the absence of preliminary investigation. agency, or instrumentality thereof, including government-owned or
controlled corporations or their subsidiaries. They shall strictly avoid
HELD: conflict of interest in the conduct of their office.
Yes, as to the first and second issuses. No, as to the third issue. CLU avers that by virtue of the phrase unless otherwise provided in
Petition was granted by the Supreme Court. this Constitution, the only exceptions against holding any other
RATIO: office or employment in Government are those provided in the
(1) The presence of a signed document bearing the signature of Constitution, namely: (i) The Vice-President may be appointed as a
Doromal as part of the application to bid shows that he can Member of the Cabinet under Sec 3, par. (2), Article 7; and (ii) the
rightfully be charged with having participated in a business which Secretary of Justice is an ex-officio member of the Judicial and Bar
act is absolutely prohibited by Section 13 of Article VII of the Council by virtue of Sec 8 (1), Article 8.
Constitution" because "the DITC remained a family corporation in
which Doromal has at least an indirect interest." ISSUE:
Section 13, Article VII of the 1987 Constitution provides that "the Whether or not EO 284 is constitutional.
President, Vice-President, the members of the Cabinet and their
deputies or assistants shall not... during (their) tenure, ...directly or HELD:
indirectly... participate in any business. No, it is unconstitutional. It is clear that the 1987 Constitution seeks
(2) The right of the accused to a preliminary investigation is "a to prohibit the President, Vice-President, members of the Cabinet,
substantial one." Its denial over his opposition is a "prejudicial error, their deputies or assistants from holding during their tenure
in that it subjects the accused to the loss of life, liberty, or property multiple offices or employment in the government, except in those
without due process of law" provided by the Constitution. cases specified in the Constitution itself and as above clarified with
Since the first information was annulled, the preliminary respect to posts held without additional compensation in an
investigation conducted at that time shall also be considered as void. ex-officio capacity as provided by law and as required by the
Due to that fact, a new preliminary investigation must be primary functions of their office, the citation of Cabinet members
conducted. (then called Ministers) as examples during the debate and
(3) The absence of preliminary investigation does not affect the deliberation on the general rule laid down for all appointive officials
court's jurisdiction over the case. Nor do they impair the validity of should be considered as mere personal opinions which cannot
the information or otherwise render it defective; but, if there were override the constitutions manifest intent and the peoples
no preliminary investigations and the defendants, before entering understanding thereof.
their plea, invite the attention of the court to their absence, the In the light of the construction given to Sec 13, Art 7 in relation to
court, instead of dismissing the information should conduct such Sec 7, par. (2), Art IX-B of the 1987 Constitution, EO 284 is
investigation, order the fiscal to conduct it or remand the case to unconstitutional. Ostensibly restricting the number of positions that
the inferior court so that the preliminary investigation may be Cabinet members, undersecretaries or assistant secretaries may
conducted. hold in addition to their primary position to not more than 2
WHEREFORE, the petition for certiorari and prohibition is granted. positions in the government and government corporations, EO 284
The Sandiganbayan shall immediately remand Criminal Case No. actually allows them to hold multiple offices or employment in
12893 to the Office of the Ombudsman for preliminary investigation direct contravention of the express mandate of Sec 13, Art 7 of the
and shall hold in abeyance the proceedings before it pending the 1987 Constitution prohibiting them from doing so, unless otherwise
result of such investigation. provided in the 1987 Constitution itself.

CIVIL LIBERTIES UNION V. EXEC. SEC. Public Interest Center vs Elma


194 SCRA 317 Political Law Ex Officio Officials Members of June 30, 2006, Chico-Nazario
the Cabinet Singularity of Office EO 284 *concurrent appointments, incompatible office
In July 1987, then President Corazon Aquino issued Executive Order N: CPM + TRO to declare null and void the concurrent appointments
No. 284 which allowed members of the Cabinet, their of ELMA as PCGG Chair and as Chief Presidential Legal Counsel
undersecretaries and assistant secretaries to hold other government
offices or positions in addition to their primary positions subject to F: Elma was appointed as PCGG Chair Oct 1998. Later on he was
limitations set therein. The Civil Liberties Union (CLU) assailed this appointed as CPLC (Jan 1999 during his term), but waived any
EO averring that such law is unconstitutional. The constitutionality remuneration that he may receive as CPLC.
of EO 284 is being challenged by CLU on the principal submission Supervening events: Theres actually no more controversy involved:
that it adds exceptions to Sec 13, Article 7 of the Constitution which In 2001, Elma was replaced by Sabio as PCGG. Nachura was then
provides: appointed as CPLC but pending resolution of the case, he was
Sec. 13. The President, Vice-President, the Members of the Cabinet, appointed SOLGEN.
and their deputies or assistants shall not, unless otherwise provided
in this Constitution, hold any other office or employment during Arguments: Public Interest Center
their tenure. They shall not, during said tenure, directly or indirectly CLU vs. Exec Sec: Art IX-B, Sec 7, par2 and Art VII, Sec13 are violated
6
by concurrent appointments
CPLC and PCGG Chair are incompatible offices Held:
NO if based on position. YES if based on primary functions test.
Arguments: Elma the strict prohibition under Section 13, Article VII of the 1987
As interpreted in CLU vs. Exec Sec, the mentioned consti provisions Constitution is not applicable to the PCGG Chairman nor to the CPLC,
dont cover other public officials given the rank of Secretary, as neither of them is a secretary, undersecretary, nor an assistant
Undersecretary, or Assistant Secretary. secretary, even if the former may have the same rank as the latter
His appointment falls under the exceptions in Art IX-B, Sec7 positions.
The 2 positions are not incompatible
*Review ulit CLU vs. Exec Sec: The language of Section 13, Article VII
NOTE: even if issue already moot, SC still took cognizance of the is a definite and unequivocal negation of the privilege of holding
case because the case is capable of repetition, and to serve as a multiple offices or employment.
guide to the bench. The Court cautiously allowed only two exceptions to the rule against
multiple offices:
Issue: (1) those provided for under the Constitution, such as Section 3,
whether the position of the PCGG Chairman or that of the CPLC falls Article VII, authorizing the Vice-President to become a member of
under the prohibition against multiple offices imposed by Section 7, the Cabinet; or
par. 2, Article IX-B of the 1987 Constitution (2) posts occupied by the Executive officials specified in Section 13,
Article VII without additional compensation in an ex-officio capacity
Held: as provided by law and as required by the primary functions of said
YES. officials office.
The crucial test in determining whether incompatibility exists The Court further qualified that additional duties must not only be
between two offices was laid out in People v. Green[13] - whether closely related to, but must be required by the officials primary
one office is subordinate to the other, in the sense that one office functions. Moreover, the additional post must be exercised in an
has the right to interfere with the other. ex-officio capacity, which denotes an act done in an official
character, or as a consequence of office, and without any other
[I]ncompatibility between two offices, is an inconsistency in the appointment or authority than that conferred by the office.[18]
functions of the two; x x x Where one office is not subordinate to Thus, it will not suffice that no additional compensation shall be
the other, nor the relations of the one to the other such as are received by virtue of the second appointment, it is mandatory that
inconsistent and repugnant, there is not that incompatibility from the second post is required by the primary functions of the first
which the law declares that the acceptance of the one is the appointment and is exercised in an ex-officio capacity.
vacation of the other. The force of the word, in its application to this
matter is, that from the nature and relations to each other, of the *Even Section 13, Article VII does not sanction this dual
two places, they ought not to be held by the same person, from the appointment. Appointment to the position of PCGG Chairman is not
contrariety and antagonism which would result in the attempt by required by the primary functions of the CPLC, and vice versa.
one person to faithfully and impartially discharge the duties of one,
toward the incumbent of the other. x x x The offices must In sum, the prohibition in Section 13, Article VII of the 1987
subordinate, one [over] the other, and they must, per se, have the Constitution does not apply to respondent Elma since neither the
right to interfere, one with the other, before they are incompatible PCGG Chairman nor the CPLC is a Cabinet secretary, undersecretary,
at common law. x x x or assistant secretary. Even if this Court assumes, arguendo, that
***In this case, an incompatibility exists between the positions of Section 13, Article VII is applicable to respondent Elma, he still could
the PCGG Chairman and the CPLC. The duties of the CPLC include not be appointed concurrently to the offices of the PCGG Chairman
giving independent and impartial legal advice on the actions of the and CPLC because neither office was occupied by him in an
heads of various executive departments and agencies and to review ex-officio capacity, and the primary functions of one office do not
investigations involving heads of executive departments and require an appointment to the other post. Moreover, even if the
agencies, as well as other Presidential appointees. The PCGG is, appointments in question are not covered by Section 13, Article VII
without question, an agency under the Executive Department. Thus, of the 1987 Constitution, said appointments are still prohibited
the actions of the PCGG Chairman are subject to the review of the under Section 7, Article IX-B, which covers all appointive and
CPLC. elective officials, due to the incompatibility between the primary
*note: Memorandum Order No. 152, issued on 9 July 2004 functions of the offices of the PCGG Chairman and the CPLC.
(provides that CPLC review Decision on investigation involving
Cabinet Secretaries, agency heads, or Presidential appointees with
the rank of Secretary conducted by the Presidential Anti-Graft Executive Power (includes Residual Power)
Commission (PAGC))
MARCOS VS MANGLAPUS
Issue: G.R. No. 88211 September 15 1989
whether such appointments violate the other constitutional FACTS:
provision regarding multiple offices, Section 13, Article VII of the Former President Marcos, after his and his family spent three year
1987 Constitution exile in Hawaii, USA, sought to return to the Philippines. The call is
7
about to request of Marcos family to order the respondents to issue the Philippines is to deny them not only the inherent right of
travel order to them and to enjoin the petition of the President's citizens to return to their country of birth but also the protection of
decision to bar their return to the Philippines. the Constitution and all of the rights guaranteed to Filipinos under
the Constitution;
ISSUE: 2. the President has no power to bar a Filipino from his own country;
Whether or not, in the exercise of the powers granted by the if she has, she had exercised it arbitrarily; and
Constitution, the President may prohibit the Marcoses from 3. there is no basis for barring the return of the family of former
returning to the Philippines. President Marcos. Thus, petitioners prayed that the Court
reconsider its decision, order respondents to issue the necessary
RULING: travel documents to enable Mrs. Imelda R. Marcos, Ferdinand R.
Yes According to Section 1, Article VII of the 1987 Constitution: "The Marcos, Jr., Irene M. Araneta, Imee M. Manotoc, Tommy Manotoc
executive power shall be vested in the President of the Philippines." and Gregorio Araneta to return to the Philippines, and enjoin
The phrase, however, does not define what is meant by executive respondents from implementing President Aquino's decision to bar
power although the same article tackles on exercises of certain the return of the remains of Mr. Marcos, and the other petitioners,
powers by the President such as appointing power during recess of to the Philippines.
the Congress (S.16), control of all the executive departments, 4. Commenting on the motion for reconsideration, the Solicitor
bureaus, and offices (Section 17), power to grant reprieves, General argued that the motion for reconsideration is moot and
commutations, and pardons, and remit fines and forfeitures, after academic as to the deceased Mr. Marcos. Moreover, he asserts that
conviction by final judgment (Section 19), treaty making power "the 'formal' rights being invoked by the Marcoses under the label
(Section 21), borrowing power (Section 20), budgetary power 'right to return', including the label 'return of Marcos' remains, is in
(Section 22), informing power (Section 23). reality or substance a 'right' to destabilize the country, a 'right' to
The Constitution may have grant powers to the President, it cannot hide the Marcoses' incessant shadowy orchestrated efforts at
be said to be limited only to the specific powers enumerated in the destabilization." [Comment, p. 29.] Thus, he prays that the Motion
Constitution. Whatever power inherent in the government that is for Reconsideration be denied for lack of merit.
neither legislative nor judicial has to be executive. We deny the motion for reconsideration.
1. It must be emphasized that as in all motions for reconsideration,
the burden is upon the movants, petitioner herein, to show that
G.R. No. 88211 October 27, 1989 there are compelling reasons to reconsider the decision of the
FERDINAND E. MARCOS, IMELDA R. MARCOS, FERDINAND R. Court.
MARCOS. JR., IRENE M. ARANETA, IMEE M. MANOTOC, TOMAS 2. After a thorough consideration of the matters raised in the
MANOTOC, GREGORIO ARANETA, PACIFICO E. MARCOS, NICANOR motion for reconsideration, the Court is of the view that no
YIGUEZ and PHILIPPINE CONSTITUTION ASSOCIATION compelling reasons have been established by petitioners to warrant
(PHILCONSA), represented by its President, CONRADO F. ESTRELLA, a reconsideration of the Court's decision.
petitioners, vs. HONORABLE RAUL MANGLAPUS, CATALINO The death of Mr. Marcos, although it may be viewed as a
MACARAIG, SEDFREY ORDOEZ, MIRIAM DEFENSOR SANTIAGO, supervening event, has not changed the factual scenario under
FIDEL RAMOS, RENATO DE VILLA, in their capacity as Secretary of which the Court's decision was rendered. The threats to the
Foreign Affairs, Executive Secretary, Secretary of Justice, government, to which the return of the Marcoses has been viewed
Immigration Commissioner, Secretary of National Defense and to provide a catalytic effect, have not been shown to have ceased.
Chief of Staff, respectively, respondents. On the contrary, instead of erasing fears as to the destabilization
In its decision dated September 15,1989, the Court, by a vote of that will be caused by the return of the Marcoses, Mrs. Marcos
eight (8) to seven (7), dismissed the petition, after finding that the reinforced the basis for the decision to bar their return when she
President did not act arbitrarily or with grave abuse of discretion in called President Aquino "illegal," claiming that it is Mr. Marcos, not
determining that the return of former President Marcos and his Mrs. Aquino, who is the "legal" President of the Philippines, and
family at the present time and under present circumstances pose a declared that the matter "should be brought to all the courts of the
threat to national interest and welfare and in prohibiting their world." [Comment, p. 1; Philippine Star, October 4, 1989.]
return to the Philippines. On September 28, 1989, former President 3. Contrary to petitioners' view, it cannot be denied that the
Marcos died in Honolulu, Hawaii. In a statement, President Aquino President, upon whom executive power is vested, has unstated
said: residual powers which are implied from the grant of executive
In the interest of the safety of those who will take the death of Mr. power and which are necessary for her to comply with her duties
Marcos in widely and passionately conflicting ways, and for the under the Constitution. The powers of the President are not limited
tranquility of the state and order of society, the remains of to what are expressly enumerated in the article on the Executive
Ferdinand E. Marcos will not be allowed to be brought to our Department and in scattered provisions of the Constitution. This is
country until such time as the government, be it under this so, notwithstanding the avowed intent of the members of the
administration or the succeeding one, shall otherwise decide. Constitutional Commission of 1986 to limit the powers of the
[Motion for Reconsideration, President as a reaction to the abuses under the regime of Mr.
p. 1; Rollo, p, 443.] Marcos, for the result was a limitation of specific power of the
On October 2, 1989, a Motion for Reconsideration was filed by President, particularly those relating to the commander-in-chief
petitioners, raising the following major arguments: clause, but not a diminution of the general grant of executive
1. to bar former President Marcos and his family from returning to power.
8
That the President has powers other than those expressly stated in implementation of this decision.
the Constitution is nothing new. This is recognized under the U.S. ACCORDINGLY, the Court resolved to DENY the Motion for
Constitution from which we have patterned the distribution of Reconsideration for lack of merit."
governmental powers among three (3) separate branches.
Article II, [section] 1, provides that "The Executive Power shall be
vested in a President of the United States of America." In Alexander The SC sustain the power of the president to create the
Hamilton's widely accepted view, this statement cannot be read as Commission but, invalidated the executive ordernevertheless on
mere shorthand for the specific executive authorizations that follow the ground of violation of equal protection.
it in [sections] 2 and 3. Hamilton stressed the difference between
the sweeping language of article II, section 1, and the conditional BIRAOGO VS PTC
language of article I, [section] 1: "All legislative Powers herein MARCH 28, 2013
granted shall be vested in a Congress of the United States . . ." G.R. No. 192935 December 7, 2010
Hamilton submitted that "[t]he [article III enumeration [in sections FACTS:
2 and 31 ought therefore to be considered, as intended merely to Pres. Aquino signed E. O. No. 1 establishing Philippine Truth
specify the principal articles implied in the definition of execution Commission of 2010 (PTC) dated July 30, 2010. PTC is a mere ad hoc
power; leaving the rest to flow from the general grant of that power, body formed under the Office of the President with the primary task
interpreted in confomity with other parts of the Constitution... to investigate reports of graft and corruption committed by
In Myers v. United States, the Supreme Court accepted third-level public officers and employees, their co-principals,
Hamilton's proposition, concluding that the federal executive, unlike accomplices and accessories during the previous administration,
the Congress, could exercise power from sources not enumerated, and to submit its finding and recommendations to the President,
so long as not forbidden by the constitutional text: the executive Congress and the Ombudsman. PTC has all the powers of an
power was given in general terms, strengthened by specific terms investigative body. But it is not a quasi-judicial body as it cannot
where emphasis was regarded as appropriate, and was limited by adjudicate, arbitrate, resolve, settle, or render awards in disputes
direct expressions where limitation was needed. . ." The language of between contending parties. All it can do is gather, collect and
Chief Justice Taft in Myers makes clear that the constitutional assess evidence of graft and corruption and make recommendations.
concept of inherent power is not a synonym for power without limit; It may have subpoena powers but it has no power to cite people in
rather, the concept suggests only that not all powers granted in the contempt, much less order their arrest. Although it is a fact-finding
Constitution are themselves exhausted by internal enumeration, so body, it cannot determine from such facts if probable cause exists as
that, within a sphere properly regarded as one of "executive' power, to warrant the filing of an information in our courts of law.
authority is implied unless there or elsewhere expressly limited. Petitioners asked the Court to declare it unconstitutional and to
[TRIBE, AMERICAN CONSTITUTIONAL LAW 158-159 (1978).] enjoin the PTC from performing its functions. They argued that: (a)
And neither can we subscribe to the view that a recognition of the E.O. No. 1 violates separation of powers as it arrogates the power of
President's implied or residual powers is tantamount to setting the the Congress to create a public office and appropriate funds for its
stage for another dictatorship. Despite petitioners' strained analogy, operation. (b) The provision of Book III, Chapter 10, Section 31 of
the residual powers of the President under the Constitution should the Administrative Code of 1987 cannot legitimize E.O. No. 1
not be confused with the power of the President under the 1973 because the delegated authority of the President to structurally
Constitution to legislate pursuant to Amendment No. 6 which reorganize the Office of the President to achieve economy,
provides: simplicity and efficiency does not include the power to create an
Whenever in the judgment of the President (Prime Minister), there entirely new public office which was hitherto inexistent like the
exists a grave emergency or a threat or imminence thereof, or Truth Commission. (c) E.O. No. 1 illegally amended the
whenever the interim Batasang Pambansa or the regular National Constitution and statutes when it vested the Truth Commission
Assembly fails or is unable to act adequately on any matter for any with quasi-judicial powers duplicating, if not superseding, those of
reason that in his judgment requires immediate action, he may, in the Office of the Ombudsman created under the 1987 Constitution
order to meet the exigency, issue the necessary decrees, orders, or and the DOJ created under the Administrative Code of 1987. (d) E.O.
letters of instruction, which shall form part of the law of the land, No. 1 violates the equal protection clause as it selectively targets for
There is no similarity between the residual powers of the President investigation and prosecution officials and personnel of the previous
under the 1987 Constitution and the power of the President under administration as if corruption is their peculiar species even as it
the 1973 Constitution pursuant to Amendment No. 6. First of all, excludes those of the other administrations, past and present, who
Amendment No. 6 refers to an express grant of power. It is not may be indictable. Respondents, through OSG, questioned the legal
implied. Then, Amendment No. 6 refers to a grant to the President standing of petitioners and argued that: 1] E.O. No. 1 does not
of the specific power of legislation. arrogate the powers of Congress because the Presidents executive
4. Among the duties of the President under the Constitution, in power and power of control necessarily include the inherent power
compliance with his (or her) oath of office, is to protect and to conduct investigations to ensure that laws are faithfully executed
promote the interest and welfare of the people. Her decision to bar and that, in any event, the Constitution, Revised Administrative
the return of the Marcoses and subsequently, the remains of Mr. Code of 1987, PD No. 141616 (as amended), R.A. No. 9970 and
Marcos at the present time and under present circumstances is in settled jurisprudence, authorize the President to create or form
compliance with this bounden duty. In the absence of a clear such bodies. 2] E.O. No. 1 does not usurp the power of Congress to
showing that she had acted with arbitrariness or with grave abuse appropriate funds because there is no appropriation but a mere
of discretion in arriving at this decision, the Court will not enjoin the allocation of funds already appropriated by Congress. 3] The Truth
9
Commission does not duplicate or supersede the functions of the limited to those specific powers under the Constitution. One of the
Ombudsman and the DOJ, because it is a fact-finding body and not a recognized powers of the President granted pursuant to this
quasi-judicial body and its functions do not duplicate, supplant or constitutionally-mandated duty is the power to create ad hoc
erode the latters jurisdiction. 4] The Truth Commission does not committees. This flows from the obvious need to ascertain facts and
violate the equal protection clause because it was validly created for determine if laws have been faithfully executed. The purpose of
laudable purposes. allowing ad hoc investigating bodies to exist is to allow an inquiry
into matters which the President is entitled to know so that he can
ISSUES: be properly advised and guided in the performance of his duties
1. WON the petitioners have legal standing to file the petitions and relative to the execution and enforcement of the laws of the land.
question E. O. No. 1;
2. WON E. O. No. 1 violates the principle of separation of powers by 2. There will be no appropriation but only an allotment or
usurping the powers of Congress to create and to appropriate funds allocations of existing funds already appropriated. There is no
for public offices, agencies and commissions; usurpation on the part of the Executive of the power of Congress to
3. WON E. O. No. 1 supplants the powers of the Ombudsman and appropriate funds. There is no need to specify the amount to be
the DOJ; 4. WON E. O. No. 1 violates the equal protection clause. earmarked for the operation of the commission because, whatever
funds the Congress has provided for the Office of the President will
RULING: be the very source of the funds for the commission. The amount
The power of judicial review is subject to limitations, to wit: (1) that would be allocated to the PTC shall be subject to existing
there must be an actual case or controversy calling for the exercise auditing rules and regulations so there is no impropriety in the
of judicial power; (2) the person challenging the act must have the funding.
standing to question the validity of the subject act or issuance;
otherwise stated, he must have a personal and substantial interest PTC will not supplant the Ombudsman or the DOJ or erode their
in the case such that he has sustained, or will sustain, direct injury respective powers. If at all, the investigative function of the
as a result of its enforcement; (3) the question of constitutionality commission will complement those of the two offices. The function
must be raised at the earliest opportunity; and (4) the issue of of determining probable cause for the filing of the appropriate
constitutionality must be the very lis mota of the case. 1. The complaints before the courts remains to be with the DOJ and the
petition primarily invokes usurpation of the power of the Congress Ombudsman. PTCs power to investigate is limited to obtaining facts
as a body to which they belong as members. To the extent the so that it can advise and guide the President in the performance of
powers of Congress are impaired, so is the power of each member his duties relative to the execution and enforcement of the laws of
thereof, since his office confers a right to participate in the exercise the land. 4. Court finds difficulty in upholding the constitutionality
of the powers of that institution. Legislators have a legal standing to of Executive Order No. 1 in view of its apparent transgression of the
see to it that the prerogative, powers and privileges vested by the equal protection clause enshrined in Section 1, Article III (Bill of
Constitution in their office remain inviolate. Thus, they are allowed Rights) of the 1987 Constitution. Equal protection requires that all
to question the validity of any official action which, to their mind, persons or things similarly situated should be treated alike, both as
infringes on their prerogatives as legislators. With regard to Biraogo, to rights conferred and responsibilities imposed. It requires public
he has not shown that he sustained, or is in danger of sustaining, bodies and institutions to treat similarly situated individuals in a
any personal and direct injury attributable to the implementation of similar manner. The purpose of the equal protection clause is to
E. O. No. 1. Locus standi is a right of appearance in a court of secure every person within a states jurisdiction against intentional
justice on a given question. In private suits, standing is governed by and arbitrary discrimination, whether occasioned by the express
the real-parties-in interest rule. It provides that every action terms of a statue or by its improper execution through the states
must be prosecuted or defended in the name of the real party in duly constituted authorities. There must be equality among equals
interest. Real-party-in interest is the party who stands to be as determined according to a valid classification. Equal protection
benefited or injured by the judgment in the suit or the party clause permits classification. Such classification, however, to be
entitled to the avails of the suit. Difficulty of determining locus valid must pass the test of reasonableness.
standi arises in public suits. Here, the plaintiff who asserts a public
right in assailing an allegedly illegal official action, does so as a The test has four requisites: (1) The classification rests on
representative of the general public. He has to show that he is substantial distinctions; (2) It is germane to the purpose of the law;
entitled to seek judicial protection. He has to make out a sufficient (3) It is not limited to existing conditions only; and (4) It applies
interest in the vindication of the public order and the securing of equally to all members of the same class. The classification will be
relief as a citizen or taxpayer. The person who impugns the regarded as invalid if all the members of the class are not similarly
validity of a statute must have a personal and substantial interest treated, both as to rights conferred and obligations imposed.
in the case such that he has sustained, or will sustain direct injury as Executive Order No. 1 should be struck down as violative of the
a result. The Court, however, finds reason in Biraogos assertion equal protection clause. The clear mandate of truth commission is
that the petition covers matters of transcendental importance to to investigate and find out the truth concerning the reported cases
justify the exercise of jurisdiction by the Court. There are of graft and corruption during the previous administration only. The
constitutional issues in the petition which deserve the attention of intent to single out the previous administration is plain, patent and
this Court in view of their seriousness, novelty and weight as manifest. Arroyo administration is but just a member of a class, that
precedents The Executive is given much leeway in ensuring that our is, a class of past administrations. It is not a class of its own. Not to
laws are faithfully executed. The powers of the President are not include past administrations similarly situated constitutes
10
arbitrariness which the equal protection clause cannot sanction. President to reorganize his office "to achieve simplicity, economy
Such discriminating differentiation clearly reverberates to label the and efficiency." The Office of the President is the nerve center of
commission as a vehicle for vindictiveness and selective retribution. the Executive Branch. To remain effective and efficient, the Office of
Superficial differences do not make for a valid classification. The PTC the President must be capable of being shaped and reshaped by the
must not exclude the other past administrations. The PTC must, at President in the manner he deems fit to carry out his directives and
least, have the authority to investigate all past administrations. The policies. After all, the Office of the President is the command post of
Constitution is the fundamental and paramount law of the nation to the President. (Emphasis supplied) Clearly, the abolition of the PAGC
which all other laws must conform and in accordance with which all and the transfer of its functions to a division specially created within
private rights determined and all public authority administered. the ODESLA is properly within the prerogative of the President
Laws that do not conform to the Constitution should be stricken under his continuing "delegated legislative authority to reorganize"
down for being unconstitutional. WHEREFORE, the petitions are his own office pursuant to E.O. 292. The President's power to
GRANTED. Executive Order No. 1 is hereby declared reorganize the Office of the President under Section 31 (2) and (3)
UNCONSTITUTIONAL insofar as it is violative of the equal protection of EO 292 should be distinguished from his power to reorganize the
clause of the Constitution. Office of the President Proper. Under Section 31 (1) of EO 292, the
President can reorganize the Office of the President Proper by
abolishing, consolidating or merging units, or by transferring
PICHAY V. OFFICE OF THE DEPUTY EXECUTIVE SECRETARY (2012) functions from one unit to another. In contrast, under Section 31 (2)
ODES no power to try and decide cases E.O. No. 13 empowering it and (3) of EO 292, the President's power to reorganize offices
is unconstitutional FACTS On November 15, 2010, President Benigno outside the Office of the President Proper but still within the Office
Simeon Aquino III issued Executive Order No. 13 (E.O. 13), of the President is limited to merely transferring functions or
abolishing the PAGC and transferring its functions to the Office of agencies from the Office of the President to Departments or gencies,
the Deputy Executive Secretary for Legal Affairs (ODESLA), more and vice versa. The distinction between the allowable organizational
particularly to its newly-established Investigative and Adjudicatory actions under Section 31(1) on the one hand and Section 31 (2) and
Division (IAD).On April 6, 2011, respondent Finance Secretary Cesar (3) on the other is crucial not only as it affects employees' tenurial
V. Purisima filed before the IAD-ODESLA a complaint-affidavit for security but also insofar as it touches upon the validity of the
grave misconduct against petitioner Prospero A. Pichay, Jr., reorganization, that is, whether the executive actions undertaken
Chairman of the Board of Trustees of the Local Water Utilities fall within the limitations prescribed under E.O. 292. When the
Administration (LWUA), as well as the incumbent members of the PAGC was created under E.O. 12, it was composed of a Chairman
LWUA Board of Trustees, namely, Renato Velasco, Susana Dumlao and two (2) Commissioners who held the ranks of Presidential
Vargas, Bonifacio Mario M. Pena, Sr. and Daniel Landingin, which Assistant II and I, respectively,9 and was placed directly "under the
arose from the purchase by the LWUA of Four Hundred Forty-Five Office of the President."10 On the other hand, the ODESLA, to
Thousand Three Hundred Seventy Seven (445,377) shares of stock which the functions of the PAGC have now been transferred, is an
of Express Savings Bank, Inc. On April 14, 2011, petitioner received office within the Office of the President Proper.11 Since both of
an Order3 signed by Executive Secretary Paquito N. Ochoa, Jr. these offices belong to the Office of the President Proper, the
requiring him and his co-respondents to submit their respective reorganization by way of abolishing the PAGC and transferring its
written explanations under oath. In compliance therewith, functions to the ODESLA is allowable under Section 31 (1) of E.O.
petitioner filed a Motion to Dismiss Ex Abundante Ad Cautelam 292 What actions does reorganization include? The Reorganization
manifesting that a case involving the same transaction and charge Did not Entail the Creation of a New, Separate and Distinct Office.
of grave misconduct entitled, "Rustico B. Tutol, et al. v. Prospero The abolition of the PAGC did not require the creation of a new,
Pichay, et al.", and docketed as OMB-C-A-10-0426-I, is already additional and distinct office as the duties and functions that
pending before the Office of the Ombudsman. pertained to the defunct anti-graft body were simply transferred to
the ODESLA, which is an existing office within the Office of the
ISSUE President Proper. The reorganization required no more than a mere
Whether E.O. 13 is unconstitutional for abrogating unto an alteration of the administrative structure of the ODESLA through
administrative office a quasi-judicial function through and E.O. and the establishment of a third division the Investigative and
not through legislative enactment by Congress. Adjudicatory Division through which ODESLA could take on the
additional functions it has been tasked to discharge under E.O. 13.
HELD Reorganization takes place when there is an alteration of the
NO. The President has Continuing Authority to Reorganize the existing structure of government offices or units therein, including
Executive Department under E.O. 292. In the case of Buklod ng the lines of control, authority and responsibility between them. It
Kawaning EIIB v. Zamora the Court affirmed that the President's involves a reduction of personnel, consolidation of offices, or
authority to carry out a reorganization in any branch or agency of abolition thereof by reason of economy or redundancy of functions.
the executive department is an express grant by the legislature by The IAD-ODESLA is a fact-finding and recommendatory body not
virtue of Section 31, Book III, E.O. 292 (the Administrative Code of vested with quasi-judicial powers. while the term "adjudicatory"
1987), "the President, subject to the policy of the Executive Office appears part of its appellation, the IAD-ODESLA cannot try and
and in order to achieve simplicity, economy and efficiency, shall resolve cases, its authority being limited to the conduct of
have the continuing authority to reorganize the administrative investigations, preparation of reports and submission of
structure of the Office of the President." The law grants the recommendations. E.O. 13 explicitly states that the IAD-ODESLA
President this power in recognition of the recurring need of every shall "perform powers, functions and duties xxx, of PAGC." Under
11
E.O. 12, the PAGC was given the authority to "investigate or hear Agency Doctrine]
administrative cases or complaints against all presidential FACTS:
appointees in the government" and to "submit its report and DENR Reg 12 Employees filed a petition for nullity of the
recommendations to the President." The IAD-ODESLA is a memorandum order issued by the Regional Exec. Director of DENR,
fact-finding and recommendatory body to the President, not having directing the immediate transfer of the DENR 12 Regional Offices
the power to settlecontroversies and adjudicate cases. As the Court from Cotabato to Koronadal City. The memorandum was issued
ruled in Cario v. Commission on Human Rights, and later reiterated pursuant to DENR Executive Order issued by the DENR Secretary.
in Biraogo v. The Philippine Truth Commission: Fact-finding is not
adjudication and it cannot be likened to the judicial function of a Issue:
court of justice, or even a quasi- judicial agency or office. The Whether or not DENR Secretary has the authority to reorganize the
function of receiving evidence and ascertaining therefrom the facts DENR Region 12 Office.
of a controversy is not a judicial function. To be considered as such,
the act of receiving evidence and arriving at factual conclusions in a RULING:
controversy must be accompanied by the authority of applying the The qualified political agency doctrine, all executive and
law to the factual conclusions to the end that the controversy may administrative organizations are adjuncts of the Executive
be decided or determined authoritatively, finally and definitively, Department, and the acts of the Secretaries of such departments,
subject to such appeals or modes of review as may be provided by performed and promulgated in the regular course of business, are,
law. The IAD-ODESLA does not encroach upon the powers and unless disapproved or reprobated by the Chief Executive, are
duties of the Ombudsman. Contrary to petitioner's contention, the presumptively the acts of the Chief Executive. It is corollary to the
IAD-ODESLA did not encroach upon the Ombudsman's primary control power of the President as provided for under Art. VII Sec. 17
jurisdiction when it took cognizance of the complaint affidavit filed of the 1987 Constitution: "The President shall have control of all the
against him notwithstanding the earlier filing of criminal and executive departments, bureaus, and offices. He shall ensure that
administrative cases involving the same charges and allegations the laws be faithfully executed." In the case at bar, the DENR
before the Office of the Ombudsman. The primary jurisdiction of Secretary can validly reorganize the DENR by ordering the transfer
the Ombudsman to investigate and prosecute cases refers to of the DENR XII Regional Offices from Cotabato City to Koronadal,
criminal cases cognizable by the Sandiganbayan and not to South Cotabato. The exercise of this authority by the DENR
administrative cases. It is only in the exercise of its primary Secretary, as an alter ego, is presumed to be the acts of the
jurisdiction that the Ombudsman may, at any time, take over the President for the latter had not expressly repudiated the same.
investigation being conducted by another investigatory agency.
Section 15 (1) of R.A. No. 6770 or the Ombudsman Act of 1989.
While the Ombudsman's function goes into the determination of Jose Mondano vs Fernando Silvosa
the existence of probable cause and the adjudication of the merits 97 Phil. 143 Political Law Control Power Revised Administrative
of a criminal accusation, the investigative authority of the IAD- Code Supervision Jose Mondano was the mayor of Mainit, Surigao.
ODESLA is limited to that of a fact-finding investigator whose A complaint was filed against him for rape and concubinage. The
determinations and recommendations remain so until acted upon information reached the Assistant Executive Secretary who ordered
by the President. Finally, petitioner doubts that the IAD-ODESLA can the governor to investigate the matter. Consequently, Governor
lawfully perform its duties as an impartial tribunal, contending that Fernando Silvosa then summoned Mondano and the latter
both the IAD-ODESLA and respondent Secretary Purisima are appeared before him. Thereafter Silvosa suspended Mondano.
connected to the President. The mere suspicion of partiality will not Mondano filed a petition for prohibition enjoining the governor
suffice to invalidate the actions of the IAD-ODESLA. Mere allegation from further proceeding. In his defense, Silvosa invoked the Revised
is not equivalent to proof. Bias and partiality cannot be presumed. Administrative Code which provided that he, as part of the
Petitioner must present substantial proof to show that the executive and by virtue o the order given by the Assistant Executive
lAD-ODES LA had unjustifiably sided against him in the conduct of Secretary, is with direct control, direction, and supervision over all
the investigation. No such evidence has been presented as to defeat bureaus and offices under his jurisdiction . . . and to that end may
the presumption of regularity m the performance of the fact-finding order the investigation of any act or conduct of any person in the
investigator's duties. The assertion, therefore, deserves scant service of any bureau or office under his Department and in
consideration. Every law has in its favor the presumption of connection therewith may appoint a committee or designate an
constitutionality, and to justify its nullification, there must be a clear official or person who shall conduct such investigations.
and unequivocal breach of the Constitution, not a doubtful and
argumentative one.39 Petitioner has failed to discharge the burden ISSUE:
of proving the illegality of E.O. 13, which IS indubitably a valid Whether or not the Governor, as agent of the Executive, can
exercise of the President's continuing authority to reorganize the exercise the power of control over a mayor.
Office of the President.
HELD:
No. (Note that Silvosa was asking as the agent of the Assistant
Power of Control Executive Secretary who ordered him to investigate Mondano).
The Constitution provides:
DENR VS DENR EMPLOYEES The President shall have control of all the executive departments,
G.R. No. 149724 [Alter ego of the President, Qualified Political bureaus, or offices, exercise general supervision over all local
12
governments as may be provided by law, and take care that the laws comes with the power to remove
be faithfully executed. Jose Villena was the then mayor of Makati in the 1930s. After
Under this constitutional provision the President has been invested investigation, the Secretary of Interior recommended the
with the power of control of all the executive departments, bureaus, suspension of Villena with the Office of the president who approved
or offices, but not of all localgovernments over which he has been the same. The Secretary then suspended Villena. Villena averred
granted only the power of general supervision as may be provided claiming that the Secretary has no jurisdiction over the matter. The
by law. The Department head as agent of the President has direct power or jurisdiction is lodged in the local government [the
control and supervision over all bureaus and offices under his governor] pursuant to sec 2188 of the Administrative Code. Further,
jurisdiction as provided for in section 79(c) of the Revised even if the respondent Secretary of the Interior has power of
Administrative Code, but he does not have the same control of local supervision over local governments, that power, according to the
governments as that exercised by him over bureaus and offices constitution, must be exercised in accordance with the provisions of
under his jurisdiction. Likewise, his authority to order the law and the provisions of law governing trials of charges against
investigation of any act or conduct of any person in the service of elective municipal officials are those contained in sec 2188 of the
any bureau or office under his department is confined to bureaus or Administrative Code as amended. In other words, the Secretary of
offices under his jurisdiction and does not extend to local the Interior must exercise his supervision over local governments, if
governments over which, as already stated, the President exercises he has that power under existing law, in accordance with sec 2188
only general supervision as may be provided by law. If the of the Administrative Code, as amended, as the latter provisions
provisions of section 79 (c) of the Revised Administrative Code are govern the procedure to be followed in suspending and punishing
to be construed as conferring upon the corresponding department elective local officials while sec 79 (C) of the Administrative Code is
head direct control, direction, and supervision over all local the genera law which must yield to the special law.
governments and that for that reason he may order the
investigation of an official of a local government for malfeasance in ISSUE:
office, such interpretation would be contrary to the provisions of Whether or not the Secretary of Interior can suspend an LGU official
par 1, sec 10, Article 7, of the 1935 Constitution. In administrative under investigation.
law supervision means overseeing or the power or authority of an
officer to see that subordinate officers perform their duties. If the HELD:
latter fail or neglect to fulfill them the former may take such action Yes.
or step as prescribed by law to make them perform their duties. There is no clear and express grant of power to the secretary to
Control, on the other hand, means the power of an officer to alter suspend a mayor of a municipality who is under investigation. On
or modify or nullify or set aside what a subordinate officer had done the contrary, the power appears lodged in the provincial governor
in the performance of his duties and to substitute the judgment of by sec 2188 of the Administrative Code which provides that The
the former for that of the latter. The Congress has expressly and provincial governor shall receive and investigate complaints made
specifically lodged the provincial supervision over municipal officials under oath against municipal officers for neglect of duty, oppression,
in the provincial governor who is authorized to receive and corruption or other form of maladministration of office, and
investigate complaints made under oath against municipal officers conviction by final judgment of any crime involving moral turpitude.
for neglect of duty, oppression, corruption or other form of The fact, however, that the power of suspension is expressly
maladministration of office, and conviction by final judgment of any granted by sec 2188 of the Administrative Code to the provincial
crime involving moral turpitude. And if the charges are serious, he governor does not mean that the grant is necessarily exclusive and
shall submit written charges touching the matter to the provincial precludes the Secretary of the Interior from exercising a similar
board, furnishing a copy of such charges to the accused either power. For instance, Villena admitted in the oral argument that the
personally or by registered mail, and he may in such case suspend President of the Philippines may himself suspend the petitioner
the officer (not being the municipal treasurer) pending action by the from office in virtue of his greater power of removal (sec. 2191, as
board, if in his opinion the charge be one affecting the official amended, Administrative Code) to be exercised conformably to law.
integrity of the officer in question. Sec 86 of the Revised Indeed, if the President could, in the manner prescribed by law,
Administrative Code adds nothing to the power of supervision to be remove a municipal official; it would be a legal incongruity if he
exercised by the Department Head over the administration of were to be devoid of the lesser power of suspension. And the
municipalities. In this case, the governor can only investigate incongruity would be more patent if, possessed of the power both
Mondano for crimes relating to Mondanos office. If the issue is not to suspend and to remove a provincial official (sec. 2078,
related to his office but involves a rime of moral turpitude (such as Administrative Code), the President were to be without the power
rape or concubinage as in this case), there must first be a final to suspend a municipal official. The power to suspend a municipal
conviction before a suspension may be issued. The point is, the official is not exclusive. Preventive suspension may be issued to give
governor must suspend a mayor not because hes acting as an agent way for an impartial investigation.
of the Executive but because of the power granted him by the
Revised Administrative Code.
Lacson-Magallanes Co., Inc. vs Jose Pao
21 SCRA 895 Political Law Delegation of Control Power to the
Jose Villena vs Secretary of the Interior Executive Secretary
67 Phil. 451 Political Law Control Power Supervision Jose Magallanes was permitted to use and occupy a land used for
Suspension of a Local Government Official Power to suspend pasture in Davao. The said land was a forest zone which was later
13
declared as an agricultural zone. Magallanes then ceded his rights to Whether or not the Secretary of Local Government, as the
Lacson-Magallanes Co., Inc. (LMC) of which he is a co-owner. Jose Presidents alter ego, can suspend and or remove local officials.
Pao was a farmer who asserted his claim over the same piece of
land. The Director of Lands denied Paos request. The Secretary HELD:
of Agriculture likewise denied his petition hence it was elevated to Yes. Ganzon is under the impression that the Constitution has left
the Office of the President. Executive Secretary Juan Pajo ruled in the President mere supervisory powers, which supposedly excludes
favor of Pao. LMC averred that the earlier decision of the Secretary the power of investigation, and denied her control, which allegedly
of Agriculture is already conclusive hence beyond appeal. He also embraces disciplinary authority. It is a mistaken impression because
averred that the decision of the Executive Secretary is an undue legally, supervision is not incompatible with disciplinary authority.
delegation of power. The Constitution, LMC asserts, does not
contain any provision whereby the presidential power of control The SC had occasion to discuss the scope and extent of the power of
may be delegated to the Executive Secretary. It is argued that it is supervision by the President over local government officials in
the constitutional duty of the President to act personally upon the contrast to the power of control given to him over executive officials
matter. of our government wherein it was emphasized that the two terms,
control and supervision, are two different things which differ one
ISSUE: from the other in meaning and extent. In administration law
Whether or not the power of control may be delegated to the supervision means overseeing or the power or authority of an
Executive Secretary. officer to see that subordinate officers perform their duties. If the
latter fail or neglect to fulfill them the former may take such action
HELD: or step as prescribed by law to make them perform their duties.
Yes. It is true that as a rule, the President must exercise his
constitutional powers in person. However, the president may Control, on the other hand, means the power of an officer to alter
delegate certain powers to the Executive Secretary at his discretion. or modify or nullify of set aside what a subordinate officer had done
The president may delegate powers which are not required by the in the performance of his duties and to substitute the judgment of
Constitution for him to perform personally. The reason for this the former for that of the latter. But from this pronouncement it
allowance is the fact that the resident is not expected to perform in cannot be reasonably inferred that the power of supervision of the
person all the multifarious executive and administrative functions. President over local government officials does not include the
The office of the Executive Secretary is an auxiliary unit which power of investigation when in his opinion the good of the public
assists the President. The rule which has thus gained recognition is service so requires.
that under our constitutional setup the Executive Secretary who
acts for and in behalf and by authority of the President has an The Secretary of Local Government, as the alter ego of the
undisputed jurisdiction to affirm, modify, or even reverse any president, in suspending Ganzon is exercising a valid power. He
order that the Secretary of Agriculture and Natural Resources, however overstepped by imposing a 600 day suspension.
including the Director of Lands, may issue. The act of the Executive
Secretary, acting as the alter ego of the President, shall remain valid
until reversed, disapproved, or reprobated by the President. In this DADOLE vs. COA
case, no reprobation was made hence the decision granting the land December 3, 2002
to Pao cannot be reversed. SUMMARY:
Sangguniang Panglungsod of Mandaue City enacted an ordinance
that granted additional monthly allowance of 1500 for judges
Power of General Supervision over LGUs assigned in their locality. However, DBM issued LBC 55 imposing
1000 as the maximum limit for such allowances. The City Auditor
Rodolfo Ganzon vs Court of Appeals subsequently disallowed the allowances for the judges and asked
200 SCRA 271 Political Law Control Power Local Government them to reimburse the amount in excess of 1000. The Judges filed
Facts: an MR but was denied by COA. SC reversed and ruled that DBM
Rodolfo Ganzon was the then mayor of Iloilo City. 10 complaints exceeded its supervisory power in imposing a limit not existing on
were filed against him on grounds of misconduct and misfeasance the LGC provision it claimed to interpret. The budget circular is void
of office. The Secretary of Local Government issued several for not being published.
suspension orders against Ganzon based on the merits of the
complaints filed against him hence Ganzon was facing about 600 DOCTRINE:
days of suspension. Ganzon appealed the issue to the CA and the CA Administrative circulars cannot go beyond the law they seek to
affirmed the suspension order by the Secretary. Ganzon asserted implement. A limit cannot be imposed through an administrative
that the 1987 Constitution does not authorize the President nor any circular when the law it is seeking to implement does not provide
of his alter ego to suspend and remove local officials; this is because for such (own words)
the 1987 Constitution supports local autonomy and strengthens the
same. What was given by the present Constitution was mere The DBM has 90 days from the receipt of the copies of the
supervisory power. appropriation ordinance of cities to review the same. Otherwise,
such ordinance was deemed to have been properly reviewed and
ISSUE: deemed to have taken effect. After such period, the legality of the
14
provisions in the said ordinance can no longer be questioned. power to inspect at any time the financial accounts of local
government units. Moreover, the Solicitor General opines that the
FACTS: DBM and the respondent are only authorized under RA 7160 to
In 1986, the RTC and MTC judges of Mandaue City started receiving promulgate a Budget Operations Manual for local government units,
monthly allowances of P1,260 each through the yearly to improve and systematize methods, techniques and procedures
appropriation ordinance enacted by the Sangguniang Panlungsod of employed in budget preparation, authorization, execution and
the said city. In 1991, Mandaue City increased the amount to P1,500 accountability pursuant to Section 354 of RA 7160. The Solicitor
for each judge. General points out that LBC 55 was not exercised under any of the
aforementioned provisions.
On March 15, 1994, the DBM issued the disputed Local Budget
Circular No. 55 (LBC 55) which provided that the additional Arguments of COA: The constitutional and statutory authority of a
allowances granted by LGUs to national government officials and city government to provide allowances to judges stationed therein is
employees assigned in their locality in the form of honorarium at not absolute. Congress may set limitations on the exercise of
rates shall not exceed P1,000.00 in provinces and cities and autonomy. It is for the President, through the DBM, to check
P700.00 in municipalities. The said circular likewise provided for its whether these legislative limitations are being followed by the local
immediate effectivity without need of publication: government units. The disbursement of additional allowances and
other benefits to judges subject to the condition that the finances
Acting on the DBM directive, the Mandaue City Auditor issued of the city government should allow the same. Thus, DBM is merely
notices of disallowance to herein petitioners, namely, Honorable enforcing the condition of the law when it sets a uniform maximum
RTC Judges Mercedes G. Dadole, Ulric R. Caete, Agustin R. Vestil, amount for the additional allowances that a city government can
Honorable MTC Judges Temistocles M. Boholst, Vicente C. Fanilag release to judges stationed therein. Also, publication is not required
and Wilfredo A. Dagatan, in excess of the amount authorized by LBC for LBC 55 inasmuch as it is merely an interpretative regulation
55. Beginning October, 1994, the additional monthly allowances of applicable to the personnel of an LGU. Assuming arguendo that LBC
the petitioner judges were reduced to P1,000 each. They were also 55 is void, the provisions of the yearly approved ordinance granting
asked to reimburse the amount they received in excess of P1,000 additional allowances to judges are still prohibited by the
from April to September, 1994. appropriation laws passed by Congress every year because
Mandaue City gets the funds for the said additional allowances of
The petitioner judges filed with the City Auditor a protest against judges from the Internal Revenue Allotment (IRA) and the General
the notices of disallowance. This was treated as a motion for Appropriations Acts of 1994 and 1995 do not mention the
reconsideration and indorsed to the COA Regional Office which in disbursement of additional allowances to judges as one of the
turn referred the motion to the head office with a recommendation allowable uses of the IRA. Hence, the provisions of said ordinance
that the same be denied. COA denied this MR ruling that the granting additional allowances, taken from the IRA, to herein
appropriation ordinance of LGUs is subject to the organizational, petitioner judges are void for being contrary to law.
budgetary and compensation policies of budgetary authorities like
the DBM who has the authority to (a) Provide guidelines on the ISSUES and RULING
classification of local government positions and on the specific rates (1) Whether LBC 55 of the DBM is void for going beyond the
of pay therefore; and (b) Provide criteria and guidelines for the supervisory powers of the President and for not having been
grant of all allowances and additional forms of compensation to published? YES, IT IS VOID.
local government employees (Sec 1, AO 42). Executive Judge (2) Whether or not the yearly appropriation ordinance enacted by
Mercedes Gozo-Dadole, for and in behalf of the petitioner judges, Mandaue City providing for fixed allowances for judges contravenes
filed a motion for reconsideration but was again denied. Hence, this any law and should therefore be struck down as null and void? NO
petition for certiorari.
RATIO:
Arguments of Judge Dadole, et al: LBC 55 is void for infringing on 1. LBC 55 is null and void.
the local autonomy of Mandaue City by dictating a uniform amount
that a local government unit can disburse as additional allowances Although our Constitution guarantees autonomy to local
to judges stationed therein. It is not supported by any law and government units, the exercise of local autonomy remains subject
therefore goes beyond the supervisory powers of the President. to the power of control by Congress and the power of supervision
Also, said circular is void for lack of publication. The yearly by the President. The supervisory power of the President has been
appropriation ordinance providing for additional allowances to interpreted to exclude the power of control. The Chief Executive
judges is allowed by Section 458, par. (a)(1)[xi], of the LGC . wielded no more authority than that of checking whether local
governments or their officials were performing their duties as
Solicitor General filed a manifestation supporting the position of the provided by the fundamental law and by statutes. He cannot
petitioner judges. The Solicitor General argues that (1) DBM only interfere with local governments, so long as they act within the
enjoys the power to review and determine whether the scope of their authority. Supervisory power, when contrasted with
disbursements of funds were made in accordance with the control, is the power of mere oversight over an inferior body; it
ordinance passed by a local government unit while (2) the COA has does not include any restraining authority over such body. (Taule v.
no more than auditorial visitation powers over local government Santos)
units pursuant to Section 348 of RA 7160 which provides for the
15
Officers in control lay down the rules in the performance or Administrative rules and regulations must also be published if their
accomplishment of an act. On the other hand, supervising officials purpose is to enforce or implement existing law pursuant to a valid
merely see to it that the rules are followed, but they themselves do delegation. (Taada vs. Tuvera). At the very least, before an
not lay down such rules, nor do they have the discretion to modify administrative circular reducing income of government officials and
or replace them. If the rules are not observed, they may order the employees should be effective, the concerned officials and
work done or redone, but only to conform to such rules. They may employees should be apprised and alerted by the publication of
not prescribe their own manner of execution of the act. They have subject circular in the Official Gazette or in a newspaper of general
no discretion on this matter except to see to it that the rules are circulation in the Philippines to the end that they be given amplest
followed. (Drilon v. Lim) opportunity to voice out whatever opposition they may have, and
to ventilate their stance on the matter. This approach is more in
Under our present system of government, executive power is vested keeping with democratic precepts and rudiments of fairness and
in the President. The members of the Cabinet and other executive transparency. (De Jesus vs. Commission on Audit). Subsequent
officials are merely alter egos. As such, they are subject to the publication will not cure the defect because the publication is
power of control of the President, at whose will and behest they can required as a condition precedent to the effectivity of a law to
be removed from office; or their actions and decisions changed, inform the public of the contents of the law or rules and regulations
suspended or reversed. In contrast, the heads of political before their rights and interests are affected by the same.
subdivisions are elected by the people. Their sovereign powers (Philippine International Trading Corporation vs. Commission on
emanate from the electorate, to whom they are directly Audit)
accountable. By constitutional fiat, they are subject to the
Presidents supervision only, not control, so long as their acts are 2. The yearly appropriation ordinance enacted by Mandaue City
exercised within the sphere of their legitimate powers. By the same providing for fixed allowances for judges is VALID.
token, the President may not withhold or alter any authority or
power given them by the Constitution and the law. Respondent COA failed to prove that Mandaue City used the IRA to
spend for the additional allowances of the judges. There was no
Clearly then, the President can only interfere in the affairs and evidence submitted by COA showing the breakdown of the
activities of a local government unit if he or she finds that the latter expenses of the city government and the funds used for said
has acted contrary to law. This is the scope of the Presidents expenses. All the COA presented were the amounts expended, the
supervisory powers over local government units. Hence, the locally generated revenues, the deficit, the surplus and the IRA
President or any of his or her alter egos cannot interfere in local received each year. Aside from these items, no data or figures were
affairs as long as the concerned local government unit acts within presented to show that Mandaue City deducted the subject
the parameters of the law and the Constitution. Any directive allowances from the IRA. In other words, just because Mandaue
therefore by the President or any of his or her alter egos seeking to Citys locally generated revenues were not enough to cover its
alter the wisdom of a law-conforming judgment on local affairs of a expenditures, did not mean that the additional allowances of
local government unit is a patent nullity because it violates the petitioner judges were taken from the IRA and not from the citys
principle of local autonomy and separation of powers of the own revenues.
executive and legislative departments in governing municipal
corporations. Moreover, the DBM neither conducted a formal review nor ordered
a disapproval of Mandaue Citys appropriation ordinances, in
LBC 55 goes beyond the law it seeks to implement. Section 458, par. accordance with the procedure outlined by Sections 326 and 327
(a)(1)(xi), of RA 7160, the law that supposedly serves as the legal of the LGC. Thus, within 90 days from receipt of the copies of the
basis of LBC 55, allows the grant of additional allowances to judges appropriation ordinance, the DBM should have taken positive action.
when the finances of the city government allow. The said Otherwise, such ordinance was deemed to have been properly
provision does not authorize setting a definite maximum limit to the reviewed and deemed to have taken effect. Inasmuch as, in the
additional allowances granted to judges. Thus, the finances of a city instant case, the DBM did not follow the appropriate procedure for
government may allow the grant of additional allowances higher reviewing the subject ordinance of Mandaue City and allowed the
than P1,000 if the revenues of the said city government exceed its 90-day period to lapse, it can no longer question the legality of the
annual expenditures. Thus, to illustrate, a city government with provisions in the said ordinance granting additional allowances to
locally generated annual revenues of P40 million and expenditures judges stationed in the said city.
of P35 million can afford to grant additional allowances of more
than P1,000 each to, say, ten judges inasmuch as the finances of the WHEREFORE, the petition is hereby GRANTED, and the assailed
city can afford it. Setting a uniform amount for the grant of decision and resolution of the Commission on Audit are hereby set
additional allowances is an inappropriate way of enforcing the aside.
criterion found in Section 458, par. (a)(1)(xi), of RA 7160. The DBM
over-stepped its power of supervision over local government units Power of Appointment
by imposing a prohibition that did not correspond with the law it Basic
sought to implement. In other words, the prohibitory nature of the
circular had no legal basis. GPI vs Springer 50 Phil 259
Facts:
Furthermore, LBC 55 is void on account of its lack of publication. This is an original action of quo warranto brought in the name of
16
the Government of the Philippine Islands against three directors of naval captain, and other officers whose appointments are vested in
the National Coal Company who were elected to their positions by him in this Constitution. He shall also appoint all other officers of
the legislative members of the committee created by Acts. Nos. the Government whose appointments are not otherwise provided
2705 and 2822. The purpose of the proceeding is to test the validity for by law, and those whom he may be authorized by law to appoint.
of the part of section 4 of Act No. 2705, as amended by section 2 of The Congress may, by law, vest the appointment of other officers
Act No. 2822, which provides that "The voting power of all such lower in rank in the President alone, in the courts, or in the heads of
stock (in the National Coal Company) owned by the Government of departments, agencies, commissions, or boards.cralaw
the Philippine Islands shall be vested exclusively in a committee The President shall have the power to make appointments during
consisting of the Governor-General, the President of the Senate, the recess of the Congress, whether voluntary or compulsory, but
and the Speaker of the House of Representatives. such appointments shall be effective only until disapproved by the
Commission on Appointments or until the next adjournment of the
Sometime in the 1900s, the National Coal Company (NCC) was Congress.
created by the Philippine Congress. The law created it (Act No. 2822) PADILLA, J:
provides that: The voting power shall be vested exclusively in a
committee consisting of the Governor-General, the President of the FACTS:
Senate, and the Speaker of the House of Representatives. (1) Petitioners contend that Mison's appointment as Commissioner
In November 1926, the Governor-General (Leonard Wood) issued of the Bureau of Customs is unconstitutional by reason of its not
E.O. No. 37 which divested the voting rights of the Senate President having been confirmed by the Commission on Appointments. The
and House Speaker in the NCC. The EO emphasized that the voting respondents, on the other hand, maintain the constitutionality of
right should be solely lodged in the Governor-General who is the respondent Mison's appointment without the confirmation of the
head of the government (President at that time was considered the Commission on Appointments.
head of state but does not manage government affairs). A copy of (2) There are four (4) groups of officers whom the President shall
the said EO was furnished to the Senate President and the House appoint.
Speaker.
However, in December 1926, NCC held its elections and the Senate First, the heads of the executive departments, ambassadors, other
President as well as the House Speaker, notwithstanding EO No. 37 public ministers and consuls, officers of the armed forces from the
and the objection of the Governor-General, still elected Milton rank of colonel or naval captain, and other officers whose
Springer and four others as Board of Directors of NCC. Thereafter, a appointments are vested in him in this Constitution;
quo warranto proceeding in behalf of the government was filed Second, all other officers of the Government whose appointments
against Springer et al questioning the validity of their election into are not otherwise provided for by law;
the Board of NCC. Third, those whom the President may be authorized by law to
appoint;
Issue: Fourth, officers lower in rank 4 whose appointments the Congress
Whether or nor EO no. 37 is invalid. may by law vest in the President alone.

Rulings: The first group of officers is clearly appointed with the consent of
No. E.O. No 37 is valid. It is in accordance with the doctrine of the Commission on Appointments.
separation of powers. The Supreme Court emphasized that the The second, third and fourth groups of officers are the present bone
legislature creates the public office but it has nothing to do with of contention.
designating the persons to fill the office. Appointing persons to a
public office is essentially executive. The NCC is a government ISSUE:
owned and controlled corporation. It was created by Congress. To Whether or not the President can appoint Mison without
extend the power of Congress into allowing it, through the Senate submitting his nomination to the Commission on Appointments
President and the House Speaker, to appoint members of the NCC is
already an invasion of executive powers. The Supreme Court HELD:
however notes that indeed there are exceptions to this rule where Petition dismissed. President of the Philippines acted within her
the legislature may appoint persons to fill public office. Such constitutional authority and power in appointing respondent
exception can be found in the appointment by the legislature of Salvador Mison, Commissioner of the Bureau of Customs, without
persons to fill offices within the legislative branch this exception is submitting his nomination to the Commission on Appointments for
allowable because it does not weaken the executive branch. confirmation. He is thus entitled to exercise the full authority and
functions of the office and to receive all the salaries and
No need of concurrence of CA emoluments pertaining thereto.

Sarmiento III v Mison In sum:


GR No. 79974 December 17, 1987 1. Confirmation by the Commission on Appointments is required
Section 16. The President shall nominate and, with the consent of only for presidential appointees mentioned in the first sentence of
the Commission on Appointments, appoint the heads of the Section 16, Article VII, including, those officers whose appointments
executive departments, ambassadors, other public ministers and are expressly vested by the Constitution itself in the president (like
consuls, or officers of the armed forces from the rank of colonel or sectoral representatives to Congress and members of the
17
constitutional commissions of Audit, Civil Service and Election). - Malilin invoked EO 163-A which provides that the tenure of the
2. Confirmation is not required when the President appoints other Chairman and the Commissioners of the CHR should be at the
government officers whose appointments are not otherwise pleasure of the President thus stating that Bautista shall be
provided for by law or those officers whom he may be authorized by subsequently removed as well.
law to appoint (like the Chairman and Members of the Commission
on Human Rights). Also, as observed in Mison, when Congress Issues:
creates inferior offices but omits to provide for appointment thereto, WON the President's appointment is considered constitutional.
or provides in an unconstitutional manner for such appointments, WON or not Bautista's appointment is subject to CoA's
the officers are considered as among those whose appointments confirmation.
are not otherwise provided for by law. WON or not President should extend her appointment on January
14, 1989.
RATIO:
(1) By following the accepted rule in constitutional and statutory Held:
construction that an express enumeration of subjects excludes Sec. 16, Art. VII of the 1987 Constitution provides:
others not enumerated, it would follow that only those The President shall nominate and, with the consent of the
appointments to positions expressly stated in the first group require Commission on Appointments, appoint the heads of the executive
the consent (confirmation) of the Commission on Appointments. departments, ambassadors, other public ministers and consuls, or
(2) It is evident that the position of Commissioner of the Bureau of officers of the armed forces from the rank of colonel or naval
Customs (a bureau head) is not one of those within the first group captain, and other officers whose appointments are vested in him in
of appointments where the consent of the Commission on this Constitution. He shall also appoint all other officers of the
Appointments is required. As a matter of fact, as already pointed Government whose appointments are not otherwise provided for
out, while the 1935 Constitution includes "heads of bureaus" by law, and those whom he may be authorized by law to appoint.
among those officers whose appointments need the consent of the
Commission on Appointments, the 1987 Constitution, on the other The Congress may, by law, vest the appointment of other officers
hand, deliberately excluded the position of "heads of bureaus" from lower in rank in the President alone, in the courts, or in the heads of
appointments that need the consent (confirmation) of the the departments, agencies, commissions or boards. The President
Commission on Appointments. shall have the power to make appointments during the recess of the
Congress, whether voluntary or compulsory, but such appointments
(3) In the 1987 Constitution, however, as already pointed out, the shall be effective only until disapproval by the Commission on
clear and expressed intent of its framers was to exclude presidential Appointments or until the next adjournment of the Congress.
appointments from confirmation by the Commission on
Appointments, except appointments to offices expressly mentioned The Court held that it is within the authority of the President,
in the first sentence of Sec. 16, Art. VII. Consequently, there was no vested upon her by the Constitution, that she appoint Executive
reason to use in the third sentence of Sec. 16, Article VII the word officials. The second sentence of the provision Section 16, Article VII
"alone" after the word "President" in providing that Congress may provides that the President is authorized by law to appoint, without
by law vest the appointment of lower-ranked officers in the confirmation of CoA, several government officials. The position of
President alone, or in the courts, or in the heads of departments, Chairman of CHR is not among the positions mentioned in the first
because the power to appoint officers whom he (the president) may sentence of Sec. 16, Art VII of the 1987 Constitution, which provides
be authorized by law to appoint is already vested in the President, the appointments which are to be made with the confirmation of
without need of confirmation by the Commission on Appointments, CoA. It therefore follows that the appointment of the Chairman of
in the second sentence of the same Sec. 16, Article VII. CHR by the President is to be made and finalized even without the
review or participation of CoA. Bautista's appointment as the
Chairman of CHR, therefore, was already a completed act on the
Bautista Vs. Salonga day she took her oath as the appointment was finalized upon her
Facts: acceptance, expressly stated in her oath.
- August 27, 1987: President Cory Aquino appointed petitioner Mary
Concepcion Bautista as permanent Chairman of the Commission on Furthermore, the Court held that the provisions of EO 163-A is
Human Rights (CHR). unconstitutional and thus cannot be invoked by Mallillin. The
- December 22, 1988: Bautista took her oath of office to Chief Chairman of CHR cannot be removed at the pleasure of the
Justice Marcelo Fernan and immediately acted as such. President for it is constitutionally guaranteed that they must have a
- January 9, 1989: The Secretary of the Commission on term of office.
Appointments (CoA) wrote a letter to Bautista requesting for her
presence along with several documents at the office of CoA on To hold, as the Court holds, that petitioner Bautista is the lawful
January 19. Bautista refused to be placed under CoA's review. incumbent of the office of Chairman of the Commission on Human
- Bautista filed a petition with the Supreme Court. Rights by virtue of her appointment, as such, by the President on 17
- While waiting for the progress of the case, President Aquino December 1988, and her acceptance thereof, is not to say that she
appointed Hesiquio R. Mallillin as "Acting Chairman of the cannot be removed from office before the expiration of her seven (7)
Commission on Human Rights" but he was not able to sit in his year term. She certainly can be removed but her removal must be
appointive office because of Bautista's refusal to surrender her post. for cause and with her right to due process properly safeguarded.
18
unconstitutional in the light of Section 16 of Article VII of the
It is to the credit of the President that, in deference to the rule of Constitution.
law, after petitioner Bautista had elevated her case to this Tribunal,
Her Excellency merely designated an Acting Chairman for the The Courts Ruling
Commission on Human Rights (pending decision in this case) instead
of appointing another permanent Chairman. The latter course At the outset, the Court recognized the occurrence of a supervening
would have added only more legal difficulties to an already difficult event that could have rendered the case moot the resignation of
situation. the Rufino group and the appointment of new CCP trustees by
President Gloria Macapagal-Arroyo. The Court, however, deemed it
Petitioner Bautista is declared to be, as she is, the duly appointed best to pass upon the merits of the case, in order to prevent a
Chairman of the Commission on Human Rights and the lawful repeat of this regrettable controversy and to protect the CCP from
incumbent thereof, entitled to all the benefits, privileges and being periodically wracked by internecine politics. Moreover, the
emoluments of said office. The temporary restraining order Court brushed aside procedural barriers, in view of the paramount
heretofore issued by the Court against respondent Mallillin importance of the constitutional issues involved.
enjoining him from dismissing or terminating personnel of the By a vote of 10-3,[2] the Court held that Section 6 (b and c) of PD 15
Commission on Human Rights is made permanent. was irreconcilable with Section 16 of Article VII of the Constitution.
The clear and categorical language of Section 6 (b) of PD 15 states
Petition granted. that vacancies in the CCP board shall be filled by a majority vote of
the remaining trustees. It is only when the board becomes entirely
Rufino V. Endriga vacant that the vacancies shall be filled by the President of the
Does the President Have the Power to Appoint CCP Trustees? Philippines, acting in consultation with the same ranking officers of
The Facts the CCP. Thus, Section 6 (b) empowers the remaining trustees of the
Petitioners in GR No. 139565, led by Baltazar N. Endriga (the Endriga board to fill the vacancies by electing their fellow trustees. Simply
group), were appointed members of the board of trustees of the put, this provision authorizes the appointing officer to appoint an
Cultural Center of the Philippines (CCP) by President Fidel V. Ramos officer who will be equal in rank to the former.
in 1995, with the qualification that their appointments would In its Decision, the Court held that the power of appointment
extend only until December 31, 1998. On December 22, 1998, then granted in Section 6 (b) of PD 15 transgressed Section 16 of Article
President Joseph Estrada advised petitioners that they were being VII of the Constitution.[3]
replaced by seven new appointees to the CCP board, led by Armita It explained that the power to appoint vested by Section 16 in the
B. Rufino (the Rufino group). Having been dislodged from the CCP, President; or the heads of departments, agencies, commissions or
Endriga filed quo warranto proceedings questioning the Presidents boards was restricted only to officers lower in rank. This
authority to appoint new members in the CCP board. constitutional provision clearly excluded a situation in which the
It was alleged that under Section 6(b)[1] of Presidential Decree No. appointing officers appointed an officer who would be equal to
15, vacancies in the board shall be filled by election by a vote of a them in
majority of the trustees held at the next regular meeting x x x. rank.
The Endriga group claimed that it was only when the board was This latter situation, however, was present in the CCP, whose
entirely vacant that the President of the Philippines may fill the trustees were appointing new co-trustees who would be equal in
vacancies, in consultation with the ranking officers of the CCP. The rank to the former. Thus, Section 6 (b and c) of PD 15 was found to
members of the group believed that since only one seat was vacant, be unconstitutional, insofar as it violated the constitutional
President Estrada could not appoint a new board. They averred that mandate that the head of the board may be authorized to appoint
presidential appointment was unjustified, since the CCP board still lower-ranking officers only.
had 10 incumbent trustees who had the statutory power to fill any Further, Section 16 of Article VII of the Constitution authorized
vacancy in the board by election. Congress to vest specifically in the heads of departments, agencies,
On May 14, 1999, the Court of Appeals (CA) granted the quo commissions, or boards and in no other person the power to
warranto Petition. It declared the Endriga group lawfully entitled appoint lower-ranked officers. The word heads referred to the
to hold office and ousted respondents from the CCP board. The CA chairpersons of the commissions or boards, not to their members,
held that Section 6(b) of Presidential Decree (PD) 15 had clearly for several reasons.
vested in the remaining members of the board the power to elect First, the 1935, the 1973, and the 1987 Constitutions made a clear
new trustees. It ruled that the President could exercise the power to distinction whenever the power to appoint lower-ranked officers
appoint only when the board was entirely vacant. was granted to the members of or the head of a collegial body.
In its appeal before this Court, the Rufino group asserted that When conferring the power of appointment to the members of that
Section 6(b) of PD 15, which authorized the CCP trustees to elect collegial body, our past and present Constitutions used the phrases
their fellow trustees, should be declared unconstitutional. The in the courts,[4] courts,[5] the Supreme Court,[6] members
provision was allegedly repugnant to Section 16 of Article VII of the of the Cabinet, 4and
Constitution, which allowed the appointment only of officers lower the Constitutional Commissions.[7]
in rank than the appointing power. Thus, if the intention was to grant to members of a commission or
board the power to appoint lower-ranked officials, Section 16 of
The Issue Article VII of the Constitution should have used the phrase in the
The lone issue was whether Section 6 (b and c) of PD 15 was commissions or boards. But in sharp contrast, this provision vested
19
the power in the heads of the departments, agencies, commissions Decision had expanded the principle of executive control in a
or boards. manner that would empower the President to make all
Second, the deliberations[8] of the present Constitution revealed appointments of officers and officials in the Executive Branch.
that the framers had intended the phrase in the heads of This expansion of executive control allegedly resulted in the
departments, agencies, commissions, or boards to be an diminution of the congressional power embodied in the
enumeration of offices whose heads may be vested by law with the Appointments clause, which was thus rendered inutile.
power to appoint lower-ranked officers. Thus, in the enumeration, In the opinion of Justice Tinga, the Appointments clause allowed
what applied to the first office applied also to the succeeding offices Congress to grant the power of appointment to the CCP board of
mentioned. trustees, which was the head of the CCP and thus superior to the
Third, all commissions or boards had chief executives who were individual trustees. Contrary to the majority opinion, he believed
their heads. Since the Constitution spoke of heads of office, and that the appointment of fellow trustees by the board would not
all commissions or boards had chief executives or heads, that word constitute the latters appointment of an officer of equal rank. For
could have referred only to the chief executives or heads of the this reason, the board of trustees as a body was superior in rank to
commissions or boards. any of its individual members.
Given that the word heads referred to the commission or board Justice Tinga cited GMCR v. Bell Telecommunications,[13] [13]
chairpersons, not members, the Court ruled that the head of the which recognized that collective or collegiate bodies outweighed or
CCP was the chairperson of the CCP board of trustees. This outranked any individual member, even if the latter was the
conclusion was further supported by the fact that Section 8 of PD presiding officer of the body. Thus, Section 6 (b) of PD 15, which
15[9] and Section 3 of the Revised Rules and Regulations[10] of the authorized the board of trustees to elect its own members, was in
CCP recognized that its board chairperson as the head of the CCP accord with the mandate of Section 16 of Article VII of the
had the power to appoint, remove, and discipline all officers, staff Constitution, according to which the heads of agencies may be
and personnel of the CCP. authorized by Congress to appoint officers of lower rank.
Pursuant to Section 16 of Article VII of the Constitution, the
chairperson of the CCP board, as the head of the CCP, was the only The Dissent also pointed out that the statutory four-year term of
officer who could be vested by law with the power to appoint respondents had not yet expired when President Estrada advised
lower-ranked officers of the CCP. Section 6 (b) of PD 15 could not them of their replacements. By ruling against them, the majority
validly grant this power of appointment to the members of the CCP allegedly sanctioned the Presidents removal of officials whose
board, as they were not the head of the CCP. terms had been fixed by law. Allegedly, this arbitrary removal could
Moreover, Section 6 (b and c) of PD 15 was found to be not be justified by the executive control clause; otherwise, the
unconstitutional, because it ran afoul of the Presidents power of President, in the guise of executive control, would be free to violate
control under Section 17 of Article VII of the Constitution.[11] It was the laws passed by Congress. And this result was clearly not
noted that the CCP was an agency that fell under the Executive intended by the said clause, according to which the President
Branch. shall ensure that the laws be faithfully executed.[14]
Under the Revised Administrative Code of 1987, any agency not The notion that executive control authorized the removal of the
placed by law or order creating them under any specific members of the CCP board at the pleasure of the President
department fell under the Office of the President.[12] Since the contravened not only the CCP charter, but the Constitution itself,
CCP did not fall under the Legislative or the Judicial Branch of not to mention our Civil Service laws that guaranteed security of
government and was not an independent constitutional or tenure.[15]
quasi-judicial body or local government unit, then the CCP The Court, in fact, explained in Ang-Angco v. Castillo[16] [16] that
necessarily fell under the Executive Branch and should be subject to the power of executive control did not extend to the power to
the Presidents control. remove an officer who was in the Executive Department. That
However, Section 6 (b and c) of PD 15, by authorizing the trustees of earlier ruling had stated that the power [of executive control]
the CCP board to fill its vacancies, insulated the CCP from political applie[d] to the exercise of control over the acts of the subordinate
influence and pressure, specifically from the President. This and not over the actor or agent himself of the act.[17]
authority made the CCP a self-perpetuating Hence, before the expiration of their terms, officials whose terms
entity, virtually outside the control of the Chief Executive. Such had been fixed by law could not be removed from office without
public office or board could not legally exist under the present cause, even by the President.
Constitution. The fixity of their terms destroyed the power of removal at
The legislature could not have validly enacted a law that would put pleasure. Since there was no showing that respondents had validly
a government office in the Executive Branch outside the control of been removed for legal cause, their removal was consequently
the President. unconstitutional.
While the charter of the CCP vested it with autonomy of policy and Further, the Dissenting Opinion discerned in the majority Decision
operation, this charter did not free it from the Presidents control. a clash between the Presidents executive control and the
As part of the Executive Branch, the CCP could not be cut off from prerogative of Congress to dictate through legislation the eligibility
that control in the guise of insulating the latter from presidential requirements and the nature and length of public officers terms of
influence. office.
The majority was criticized for inferring that the Legislative Branch
The Dissenting Opinion of Justice Tinga had no power to legislate any form of control on executive action.
In his Dissenting Opinion, Justice Tinga opined that the majority Allegedly impaired was the right of the legislature to impart public
20
offices it had created with safeguards that would ensure their attained the position of Acting Director, through regular and
independence from executive interference, should Congress deem successive promotions, in accordance with civil service rules. On
that their independence served a necessary public purpose. In June 17, 1961, he was designated Acting Director of the same
effect, said Justice Tinga, the Decision allowed the President to Bureau, and on December 13, 1961 was appointed by President
ignore or countermand statutory limitations contained in the Carlos Garcia ad interim Director. He qualified by taking the oath of
charters of GOCCs like the CCP. office on the December 23, 1961. His appointment was on
December 26, 1961, transmitted to the Commission on
Appointments, and on May 14, 1962, petitioner's ad interim
Limitations to appoint power appointment as Director of Lands was confirmed by the
Commission.
AYTONA VS CASTILLO On November 14, 1962 he received a letter from Benjamin Gozon,
4 SCRA 1 G.R. No. L-19313 January 19 1962 [Midnight Appointment] then Secretary of Agriculture and Natural Resources of the
FACTS: Macapagal administration, informing him that pursuant to a letter
On December 29, 1961, Outgoing President Carlos Garcia appointed from the Assistant Executive Secretary Bernal, served on Jorge on
petitioner Dominador Aytona as ad interim Governor of the Central November 13, his appointment was among those revoked by
Bank. Aytona took the corresponding oath. On the same day, at Administrative Order No. 2 of President Diosdado Macapagal; that
noon, President-elect Diosdado Macapagal assumed office; and on the position of Director of Lands was considered vacant; and that
the next day, he issued administrative order no. 2 recalling, Jorge was designated Acting Director of Lands, effective November
withdrawing, and cancelling all ad interim appointments made by 13, 1962. Upon learning that Mayor, an outsider, had been
former President Garcia. There were all-in all, 350 midnight or last designate by the President to be Acting Director of Lands Jorge
minute appointments made by the former President Garcia. On protested (in a letter of November 16, 1962) to the Secretary of
January 1, President Macapagal appointed Andres Castillo as ad Agriculture informing the latter that he would stand on his rights,
interim Governor of the Central Bank. Aytona instituted a case (quo and issued office circulars claiming to be the legally appointed
warranto) against Castillo, contending that he was validly appointed, Director of Lands. Finally, on September 2, 1963, he instituted the
thus the subsequent appointment to Castillo by the new President, present proceedings.
should be considered void.
ISSUE:
ISSUE: Whether or not Administrative Order No. 2 of President Macapaga
Whether or not the 350 midnight appointments of former President operated as a valid revocation of Jorge's ad interim appointment.
Garcia were valid.
RULING:
RULING: No. The SC ruled that Nicanor G. Jorge is declared to be the duly
No. After the proclamation of the election of President Macapagal, appointed, confirmed, and qualified Director of Lands.
previous President Garcia administration was no more than a Petitioner Jorge's ad interim appointment is dated December 13,
care-taker administration. He was duty bound to prepare for the 1961, but there is no evidence on record that it was made and
orderly transfer of authority the incoming President, and he should released after the joint session of Congress that ended on the same
not do acts which he ought to know, would embarrass or obstruct day. It is a matter of contemporary history, of which SC may take
the policies of his successor. It was not for him to use powers as judicial cognizance, that the session ended late in the night of
incumbent President to continue the political warfare that had December 13, 1961, and, therefore, after regular office hours. In the
ended or to avail himself of presidential prerogatives to serve absence of competent evidence to the contrary, it is to be presumed
partisan purposes. The filling up vacancies in important positions, if that the appointment of Jorge was made before the close of office
few, and so spaced to afford some assurance of deliberate action hours, that being the regular course of business. The appointment,
and careful consideration of the need for the appointment and the therefore, was not included in, nor intended to be covered by,
appointee's qualifications may undoubtedly be permitted. But the Administrative Order No. 2, and the same stands unrevoked.
issuance of 350 appointments in one night and planned induction of Consequently, it was validly confirmed by the Commission on
almost all of them a few hours before the inauguration of the new Appointments, and thereafter, the office never became vacant.
President may, with some reason, be regarded by the latter as an In common with the Gillera appointment sustained by the SC,
abuse Presidential prerogatives, the steps taken being apparently a Jorge's appointment is featured by a recognition of his tenure by the
mere partisan effort to fill all vacant positions irrespective of fitness Macapagal administration itself, since he was allowed to hold and
and other conditions, and thereby deprive the new administration discharge undisturbed his duties as de jure Director of Lands for
of an opportunity to make the corresponding appointments. nearly eleven months and it was only in mid-November of 1962 that
the attempt was actually made to demote him and appoint a rank
outsider in his place in the person of respondent Mayor.
Jorge vs Mayor If anyone is entitled to the protection of the civil service provisions
G.R. No. L-21776 February 28, 1964 of the Constitution, particularly those against removals without
FACTS: lawful cause, it must be the officers who, like Jorge, entered the
Nicanor G. Jorge, is a career official in the Bureau of Lands. He Civil Service in their youth, bent on making a career out of it, gave it
started working there as a Junior Computer in the course of 38 the best years of their lives and grew gray therein in the hope and
years service, from February 1, 1922 to October 31, 1960, and expectation that they would eventually attain the upper reaches
21
and levels of the official hierarchy, not through political patronage,
but through loyalty, merit, and faithful and unremitting toil. 2. NO. The mere fact that the Senate adjourned sine die at midnight
of January 22, 1966, the House of the Representative is only a part
of the Congress and not the Congress itself. So logically, the
Ad interim or recess appointment vs regular adjournment of one of its Houses is considered adjournment of the
Congress as a whole. And the petitioners ad interim appointment
GUEVARA VS. INOCENTES, G. R. NO. L-25577, 16 SCRA 379, MARCH must have been lapsed on January 22, 1966 upon adjournment of
15, 1966 the Senate.
FACTS:
The petitioner, Onofre Guevara was extended an ad interim
appointment as Undersecretary of Labor by the former Executive on MATIBAG VS. BENIPAYO, G.R. No. 149036, April 2, 2002
November 18, 1965. Took his oath of office on November 25th FACTS:
same year. The incumbent Executive issued Memorandum Circular
No. 8 dated January 23, 1966 declaring that all ad interim On February 1999, petitioner Maria Angelina Matibag was
appointments made by the former Executive lapsed with the appointed Acting Director IV of the Comelecs EID by then Comelec
adjournment of the special session of Congress at about midnight of Chairperson Harriet Demetriou in a temporary capacity. On March
January 22, 1966. The respondent, Raoul Inocentes was extended 2001, respondent Benipayo was appointed Comelec Chairman
an ad interim appointment for the same position by the incumbent together with other commissioners in an ad interim appointment.
Executive on January 23, 1966. Guevara filed before the court an While on such ad interim appointment, respondent Benipayo in his
instant petition for Quo Warranto seeking to be declared person capacity as Chairman issued a Memorandum address transferring
legally entitled to the said Officer of the Undersecretary of Labor petitioner to the Law Department. Petitioner requested Benipayo to
under Art. VII Sec. 10 (4) of the 1935 Constitution. which states that: reconsider her relief as Director IV of the EID and her reassignment
to the Law Department. She cited Civil Service Commission
The President shall have the power to make appointments during Memorandum Circular No. 7 dated April 10, 2001, reminding heads
the recess of the Congress, but such appointments shall be effective of government offices that transfer and detail of employees are
only until disapproval by the Commission on Appointments or until prohibited during the election period.
the next adjournment of Congress.
Benipayo denied her request for reconsideration on April 18, 2001,
Since there was no Commission on Appointments organized during citing COMELEC Resolution No. 3300 dated November 6, 2000,
the special session which commenced on January 17, 1966, the exempting Comelec from the coverage of the said Memo Circular.
respondent contended that the petitioners ad interim appointment
as well as other made under similar conditions must have lapsed Petitioner appealed the denial of her request for reconsideration to
when the Congress adjourned its last special session. But the the COMELEC en banc. She also filed an administrative and criminal
petitioner stated that (1) the specific provision in the Constitution complaint Department 17 against Benipayo, alleging that her
which states that: until the next adjournment of Congress means reassignment violated Section 261 (h) of the Omnibus Election Code,
adjournment of a regular session of Congress and not by a special COMELEC Resolution No. 3258, Civil Service Memorandum Circular
session and (2) only the Senate adjourned sine die at midnight of No. 07, s. 001, and other pertinent administrative and civil service
January 22, 1966 and the House of the Representative merely laws, rules and regulations.
suspended its session and to be resumed on January 24, 1966 at
10:00 AM. The petitioner therefore concludes that Congress has During the pendency of her complaint before the Law Department,
been in continuous session without interruption since January 17. petitioner filed the instant petition questioning the appointment
and the right to remain in office of Benipayo, Borra and Tuason, as
ISSUE/S: Chairman and Commissioners of the COMELEC, respectively.
1. Whether or not, the petitioners contention regarding the next Petitioner claims that the ad interim appointments of Benipayo,
adjournment of Congress specifically provides for regular session Borra and Tuason violate the constitutional provisions on the
only. independence of the COMELEC.
2. Whether or not, the petitioners contention that Congress is still
in continuous session? ISSUES:
Whether or not the assumption of office by Benipayo, Borra and
HELD: Tuason on the basis of the ad interim appointments issued by the
1. NO. The phrase until the next adjournment of Congress does President amounts to a temporary appointment prohibited by
not make any reference to specific session of Congress, whether Section 1 (2), Article IX-C of the Constitution.
regular or special. But a well-know Latin maxim is statutory
construction stated that when the law does not distinguish we RULING:
should not distinguish. Ubi lex non distinguit nec nos distinguere We find petitioners argument without merit.
debemus. It is safe to conclude that the authors of the 1935
Constitution used the word adjournment had in mind either An ad interim appointment is a permanent appointment because it
regular or special and not simply the regular one as the petitioner takes effect immediately and can no longer be withdrawn by the
contended. President once the appointee has qualified into office. The fact that
22
it is subject to confirmation by the Commission on Appointments Department. However, Section 4(1) and Section 9, Article VIII,
does not alter its permanent character. The Constitution itself mandate the President to fill the vacancy in the Supreme Court
makes an ad interim appointment permanent in character by within 90 days from the occurrence of the vacancy, and within 90
making it effective until disapproved by the Commission on days from the submission of the list, in the case of the lower courts.
Appointments or until the next adjournment of Congress. The 90-day period is directed at the President, not at the JBC. Thus,
the JBC should start the process of selecting the candidates to fill
the vacancy in the Supreme Court before the occurrence of the
Limitation on appointment power of acting president vacancy. The JBC has no discretion to submit the list to the
Sec 14-15 90 day ban on appointment during election period President after the vacancy occurs, because that shortens the
does not apply to: 90-day period allowed by the Constitution for the President to make
1. judicial appointment the appointment. For the JBC to do so will be unconscionable on its
part, considering that it will thereby effectively and illegally deprive
ARTURO M. DE CASTRO vs. JUDICIAL AND BAR COUNCIL (JBC) and the President of the ample time granted under the Constitution to
PRESIDENT GLORIA M. ARROYO reflect on the qualifications of the nominees named in the list of the
G.R. No. 191002, March 17, 2010 JBC before making the appointment. The duty of the JBC to submit
FACTS: a list of nominees before the start of the President's mandatory
Seven days After 2010 Presidential Election, Chief Justice Reynato S. 90-day period to appoint is ministerial, but its selection of the
Puno had his compulsory Retirement by May 17, 2010. Even before candidates whose names will be in the list to be submitted to the
the event actually happens, it is giving rise to many legal dilemmas. President lies within the discretion of the JBC. The object of the
May the incumbent President appoint his successor, considering petitions for mandamus herein should only refer to the duty to
that Section 15, Article VII (Executive Department) of the submit to the President the list of nominees for every vacancy in the
Constitution prohibits the President or Acting President from Judiciary, because in order to constitute unlawful neglect of duty,
making appointments within two months immediately before the there must be an unjustified delay in performing that duty. The
next presidential elections and up to the end of his term, except distinction between a ministerial and discretionary act is well
temporary appointments to executive positions when continued delineated. A purely ministerial act or duty is one which an officer
vacancies therein will prejudice public service or endanger public or tribunal performs in a given state of facts, in a prescribed manner,
safety. But under Section 4 (1), Article III judicial Department of the in obedience to the mandate of a legal authority, without regard to
Constitution, which provides that any vacancy in the Supreme Court or the exercise of his own judgment upon the propriety or
shall be filled within 90 days from the occurrence thereof, from a impropriety of the act done.
"list of at least three nominees prepared by the Judicial and Bar 2. Local officals
Council for every vacancy." This provision sin in contrast with the
provision mentioned above. CONRADO L. DE RAMA VS. THE COURT OF APPEALS (NINTH
DIVISION, THE CIVIL SERVICE COMMISSION), G.R. NO. 131136.
The question now arises whether the incumbent president has the FEBRUARY 28, 2001
right to appoint the next chief Justice upon the retirement of Chief Facts:
Justice Puno. The JBC has unanimously agreed in their meeting on Upon his assumption to the position of Mayor of Pagbilao, Quezon,
January 18, 2011, to start the process of the filling the Vacant petitioner Conrado L. de Rama wrote a letter dated July 13, 1995 to
position of the Retired Chief Justice. Judicial Bar Council has the Civil Service Commission (or CSC), seeking the recall of the
published the said announcement in the Daily inquirer and appointments of fourteen (14) municipal employees. Justifying his
Philippine Star news papers on January 20, 2010. Despite of the recall request on the allegation that the appointments of the said
issues JBC has decided to proceed to the next step of the process by employees were midnight appointments of the former mayor, Ma.
announcing the names of the following Associate Justices: Associate Evelyn S. Abeja, done in violation of Article VII, Section 15 of the
Justice Carpio, Associate Justice Corona, Associate Justice Carpio 1987 Constitution. The CSC denied petitioners request for the recall
Morales, Associate Justice Leonardo-De Castro, Associate Justice of the appointments of the fourteen employees, for lack of merit.
Brion, and Associate Justice Sandoval and inviting the public to file The CSC upheld the validity of the appointments on the ground that
their sworn complaint, written report, or opposition, if any, not later they had already been approved by the Head of the CSC Field Office
than February 22, 2010, to the Following Candidates. The in Lucena City, and for petitioners failure to present evidence that
announcement was printed at Daily Inquirer and Philippine Star on would warrant the revocation or recall of the said appointments.
February 13, 2010.
Issue:
ISSUE: whether or not the recall made by petitioner is valid.
Whether or not Section 15, Article VII apply to appointments in the
Supreme Court or to the Judiciary. Ruling:
No. It is the CSC that is authorized to recall an appointment initially
RULING: approved, but only when such appointment and approval are
No. Prohibition under Section 15, Article VII does not apply to proven to be in disregard of applicable provisions of the civil service
appointments to fill a vacancy in the Supreme Court or to other law and regulations. Rule V, Section 9 of the Omnibus Implementing
appointments to the Judiciary the Constitutional Commission Regulations of the Revised Administrative Code specifically provides
confined the prohibition to appointments made in the Executive that an appointment accepted by the appointee cannot be
23
withdrawn or revoked by the appointing authority and shall remain However, upon the recommendation of petitioner Emilio
in force and in effect until disapproved by the Commission. Gonzales III, a Decision finding P/S Insp. Rolando Mendoza and his
Accordingly, the appointments of the private respondents may only fellow police officers guilty of Grave Misconduct was approved by
be recalled on the following grounds: (a) Non-compliance with the the Ombudsman
procedures/criteria provided in the agencys Merit Promotion Plan; They filed a Motion for Reconsideration followed by a
(b) Failure to pass through the agencys Selection/Promotion Board; Supplement to the Motion for Reconsideration. The pleadings
(c) Violation of the existing collective agreement between mentioned and the records of the case were assigned for review
management and employees relative to promotion; or (d) Violation and recommendation to Graft Investigation and Prosecutor Officer
of other existing civil service law, rules and regulations. Dennis L. Garcia, who released a draft Order for appropriate action
by his immediate superior, Director Eulogio S. Cecilio, who, in turn,
signed and forwarded said Order to petitioner Gonzalez's office on
Power of removal April 27, 2010. Not more than ten (10) days after, more particularly
on May 6, 2010, petitioner endorsed the Order, together with the
Gonzales III v Office of the President case records, for final approval by Ombudsman Merceditas N.
FACTS: Gutierrez, in whose office it remained pending for final review and
There are two petitions that have been consolidated because action when P/S Insp. Mendoza hijacked a bus-load of foreign
they raise a common thread of issues relating to the President's tourists on that fateful day of August 23, 2010 in a desperate
exercise of the power to remove from office herein petitioners who attempt to have himself reinstated in the police service.
claim the protective cloak of independence of the Incident Investigation and Review Committee (IIRC): found
constitutionally-created office to which they belong - the Office of Deputy Ombudsman Gonzales committed serious and inexcusable
the Ombudsman. negligence and gross violation of their own rules of procedure by
1st case -> G.R. No. 19623: Petition for Certiorari which assails allowing Mendoza's motion for reconsideration to languish for more
on jurisdictional grounds the Decision dated March 31, 2011 than nine (9) months without any justification, in violation of the
rendered by the Office of the dismissing petitioner Emilio A. Ombudsman prescribed rules to resolve motions for reconsideration
Gonzales III, Deputy Ombudsman for the Military and Other Law in administrative disciplinary cases within five (5) days from
Enforcement Offices, upon a finding of guilt on the administrative submission. The inaction is gross, considering there is no opposition
charges of Gross Neglect of Duty and Grave Misconduct constituting thereto. The prolonged inaction precipitated the desperate resort to
a Betrayal of Public Trust. The petition primarily seeks to declare as hostage-taking
unconstitutional Section 8(2) of Republic Act (R.A.) No. 6770, Case was elevated to OP. OP instituted a Formal Charge against
otherwise known as the Ombudsman Act of 1989, which gives the petitioner Gonzales for Gross Neglect of Duty and/or Inefficiency in
President the power to dismiss a Deputy Ombudsman of the Office the Performance of Official Duty under Rule XIV, Section 22 of the
of the Ombudsman. Omnibus Rules Implementing Book V of E.O. No. 292 and other
2nd case -> G.R. No. 196232, is a Petition for Certiorari and pertinent Civil Service Laws, rules and regulations, and for
Prohibition seeking to annul, reverse and set aside (1) the undated Misconduct in Office under Section 3 of the Anti-Graft and Corrupt
Order requiring petitioner Wendell Barreras-Sulit to submit a Practices Act.
written explanation with respect to alleged acts or omissions OP Dismissed Gonzales from his office.
constituting serious/grave offenses in relation to the Plea Bargaining 2nd case: the Acting Deputy Special Prosecutor of the Office of
Agreement entered into with Major General Carlos F. Garcia; and (2) the Ombudsman charged Major General Carlos F. Garcia, his wife
the April 7, 2011 Notice of Preliminary Investigation, both issued Clarita D. Garcia, their sons Ian Carl Garcia, Juan Paulo Garcia and
by the Office of the President the administrative case initiated Timothy Mark Garcia and several unknown persons with Plunder
against petitioner as a Special Prosecutor of the Office of the and Money Laundering before the Sandiganbayan.
Ombudsman. The petition likewise seeks to declare as
unconstitutional Section 8(2) of R.A. No. 6770 giving the President Issues:
the power to dismiss a Special Prosecutor of the Office of the Whether the Office of the President has jurisdiction to exercise
Ombudsman. administrative disciplinary power over a Deputy Ombudsman and a
Cause of 1st case: Hostage Drama involving Rolando Mendoza Special Prosecutor who belong to the constitutionally-created Office
and Hong Kong nationals in a tourist bus. Rolando Mendoza of the Ombudsman.
demanded his reinstatement. Sometime in 2008, a formal charge
for Grave Misconduct (robbery, grave threats, robbery extortion and Ruling:
physical injuries) was filed against him and other police officers. By granting express statutory
Office of the Regional Director of the National Police power to the President to remove
Commission turned over, upon the request of petitioner Emilio A. a Deputy Ombudsman and a
Gonzales III, all relevant documents and evidence in relation to said Special Prosecutor, Congress
case to the Office of the Deputy Ombudsman for appropriate merely filled an obvious gap in
administrative adjudication the law.
The administrative case against Mendoza was dismissed upon Section 9, Article XI of the 1987 Constitution confers upon the
a finding that the material allegations made by the complainant had President the power to appoint the Ombudsman and his Deputies,
not been substantiated "by any evidence at all to warrant the viz:
indictment of respondents of the offenses charged. Section 9. The Ombudsman and his Deputies shall be appointed by
24
the President from a list of at least six nominees prepared by the "reasonably insulated from the whims of politicians."52 And so it
Judicial and Bar Council, and from a list of three nominees for every was that Section 5, Article XI of the 1987 Constitution had declared
vacancy thereafter. Such appointments shall require no the creation of the independent Office of the Ombudsman,
confirmation. All vacancies shall be filled within three months after composed of the Ombudsman and his Deputies, who are described
they occur. as "protectors of the people" and constitutionally mandated to act
While the removal of the Ombudsman himself is also expressly promptly on complaints filed in any form or manner against public
provided for in the Constitution, which is by impeachment under officials or employees of the Government Section 12, Article XI.
Section 244 of the same Article, there is, however, no constitutional Pertinent provisions under Article XI prescribes a term of office of
provision similarly dealing with the removal from office of a Deputy seven years without reappointment Section 11, prohibits a decrease
Ombudsman, or a Special Prosecutor, for that matter. By enacting in salaries during the term of office Section 10, provides strict
Section 8(2) of R.A. 6770, Congress simply filled a gap in the law qualifications for the office Section 8, grants fiscal autonomy
without running afoul of any provision in the Constitution or Section 14 and ensures the exercise of constitutional functions
existing statutes. In fact, the Constitution itself, under Section 2, Section 12 and 13. The cloak of independence is meant to build up
authorizes Congress to provide for the removal of all other public the Office of the Ombudsman's institutional strength to effectively
officers, including the Deputy Ombudsman and Special Prosecutor, function as official critic, mobilizer of government, constitutional
who are not subject to impeachment. watchdog53 and protector of the people. It certainly cannot be
The Power of the President to Remove a Deputy Ombudsman and a made to extend to wrongdoings and permit the unbridled acts of its
Special Prosecutor is Implied from his Power to Appoint. officials to escape administrative discipline.
Under the doctrine of implication, the power to appoint carries with Being aware of the constitutional imperative of shielding the Office
it the power to remove.48 As a general rule, therefore, all officers of the Ombudsman from political influences and the discretionary
appointed by the President are also removable by him.49 The acts of the executive, Congress laid down two restrictions on the
exception to this is when the law expressly provides otherwise - that President's exercise of such power of removal over a Deputy
is, when the power to remove is expressly vested in an office or Ombudsman, namely: (1) that the removal of the Deputy
authority other than the appointing power. In some cases, the Ombudsman must be for any of the grounds provided for the
Constitution expressly separates the power to remove from the removal of the Ombudsman and (2) that there must be observance
President's power to appoint. Under Section 9, Article VIII of the of due process. Reiterating the grounds for impeachment laid down
1987 Constitution, the Members of the Supreme Court and judges in Section 2, Article XI of the 1987 Constitution, paragraph 1 of
of lower courts shall be appointed by the President. However, Section 8 of R.A. No. 6770 states that the Deputy Ombudsman may
Members of the Supreme Court may be removed after be removed from office for the same grounds that the Ombudsman
impeachment proceedings initiated by Congress (Section 2, Article may be removed through impeachment, namely, "culpable violation
XI), while judges of lower courts may be removed only by the of the Constitution, treason, bribery, graft and corruption, other
Supreme Court by virtue of its administrative supervision over all its high crimes, or betrayal of public trust." Thus, it cannot be rightly
personnel (Sections 6 and 11, Article VIII). The Chairpersons and said that giving the President the power to remove a Deputy
Commissioners of the Civil Service Commission Section 1(2), Article Ombudsman, or a Special Prosecutor for that matter, would
IX(B), the Commission on Elections Section 1(2), Article IX(C), and diminish or compromise the constitutional independence of the
the Commission on Audit Section 1(2), Article IX(D) shall likewise be Office of the Ombudsman. It is, precisely, a measure of protection of
appointed by the President, but they may be removed only by the independence of the Ombudsman's Deputies and Special
impeachment (Section 2, Article XI). As priorly stated, the Prosecutor in the discharge of their duties that their removal can
Ombudsman himself shall be appointed by the President (Section 9, only be had on grounds provided by law.
Article XI) but may also be removed only by impeachment (Section
2, Article XI).
In giving the President the power to remove a Deputy Ombudsman Pardoning Power
and Special Prosecutor, Congress simply laid down in express terms
an authority that is already implied from the President's PEOPLE OF THE PHILIPPINES VS. JOSE VERA, 65 PHIL. 56
constitutional authority to appoint the aforesaid officials in the Political Law Constitutional Law Bill of Rights Equal Protection
Office of the Ombudsman. Probation Law
Granting the President the Power to Remove a Deputy Ombudsman Separation of Powers Undue Delegation of Powers Power to
does not Diminish the Independence of the Office of the Pardon
Ombudsman. Constitutionality of Laws May the State Question Its Own Laws
The claim that Section 8(2) of R.A. No. 6770 granting the President In 1934, Mariano Cu Unjieng was convicted in a criminal case filed
the power to remove a Deputy Ombudsman from office totally against him by the Hongkong and Shanghai Banking Corporation
frustrates, if not resultantly negates the independence of the Office (HSBC). In 1936, he filed for probation. The matter was referred to
of the Ombudsman is tenuous. The independence which the Office the Insular Probation Office which recommended the denial of Cu
of the Ombudsman is vested with was intended to free it from Unjiengs petition for probation. A hearing was set by Judge Jose
political considerations in pursuing its constitutional mandate to be Vera concerning the petition for probation. The Prosecution
a protector of the people. What the Constitution secures for the opposed the petition. Eventually, due to delays in the hearing, the
Office of the Ombudsman is, essentially, political independence. Prosecution filed a petition for certiorari with the Supreme Court
This means nothing more than that "the terms of office, the salary, alleging that courts like the Court of First Instance of Manila (which
the appointments and discipline of all persons under the office" are is presided over by Judge Vera) have no jurisdiction to place accused
25
like Cu Unjieng under probation because under the law (Act No. commission which will act arbitrarily according to its whims.
4221 or The Probation Law), probation is only meant to be applied
in provinces with probation officers; that the City of Manila is not a Encroachment of Executive Power
province, and that Manila, even if construed as a province, has no Though Act 4221 is unconstitutional, the Supreme Court recognized
designated probation officer hence, a Manila court cannot grant the power of Congress to provide for probation. Probation does not
probation. encroach upon the Presidents power to grant pardon. Probation is
Meanwhile, HSBC also filed its own comment on the matter alleging not pardon. Probation is within the power of Congress to fix
that Act 4221 is unconstitutional for it violates the constitutional penalties while pardon is a power of the president to commute
guarantee on equal protection of the laws. HSBC averred that the penalties.
said law makes it the prerogative of provinces whether or nor to
apply the probation law if a province chooses to apply the
probation law, then it will appoint a probation officer, but if it will LLAMAS vs. EXECUTIVE SECRETARY OSCAR ORBOS and MARIANO
not, then no probation officer will be appointed hence, that UN OCAMPO III, October 15, 1991 (G.R. No. 99031)
makes it violative of the equal protection clause. FACTS:
Further, HSBC averred that the Probation Law is an undue Ocampo III was the governor of Tarlac Province. Llamas together
delegation of power because it gave the option to the provincial with some other complainants filed an administrative case against
board to whether or not to apply the probation law however, the Ocampo III for alleged acts constituting graft and corruption.
legislature did not provide guidelines to be followed by the Ocampo III was found guilty. He was suspended for office for 90
provincial board. days hence his vice governor, Llamas, assumed office. In not less
Further still, HSBC averred that the Probation Law is an than 30 days however, Ocampo III returned with an AO showing
encroachment of the executives power to grant pardon. They say that he was pardoned hence he can resume office without
that the legislature, by providing for a probation law, had in effect completing the 90 day suspension imposed upon him.
encroached upon the executives power to grant pardon. (Ironically,
the Prosecution agreed with the issues raised by HSBC ironic The petitioner argues that President may grant executive clemency
because their main stance was the non-applicability of the only in criminal cases. They say that the qualifying phrase after
probation law only in Manila while recognizing its application in conviction by final judgment applies solely to criminal cases, and
provinces). no other law allows the grant of executive clemency or pardon to
For his part, one of the issues raised by Cu Unjieng is that, the anyone who has been convicted in an administrative case, allegedly
Prosecution, representing the State as well as the People of the because the word conviction refers only to criminal cases.
Philippines, cannot question the validity of a law, like Act 4221,
which the State itself created. Further, Cu Unjieng also castigated ISSUE:
the fiscal of Manila who himself had used the Probation Law in the WON the President of the Philippines has the power to grant
past without question but is now questioning the validity of the said executive clemency in administrative cases.
law (estoppel).
HELD:
ISSUE: Yes. It is not specified in the constitution whether it may be
1. May the State question its own laws? considered under criminal or administrative cases. , if the law does
2. Is Act 4221 constitutional? not distinguish, so we must not distinguish. The Constitution does
not distinguish between which cases executive clemency may be
HELD: exercised by the President, with the sole exclusion of impeachment
1. Yes. There is no law which prohibits the State, or its duly cases. By the same token, if executive clemency may be exercised
authorized representative, from questioning the validity of a law. only in criminal cases, it would indeed be unnecessary to provide for
Estoppel will also not lie against the State even if it had been using the exclusion of impeachment cases from the coverage of Article VII,
an invalid law. Section 19 of the Constitution. Cases of impeachment are
2. No, Act 4221 or the [old] Probation Law is unconstitutional. automatically excluded inasmuch as the same do not necessarily
Violation of the Equal Protection Clause involve criminal offenses.
The contention of HSBC and the Prosecution is well taken on this
note. There is violation of the equal protection clause. Under Act The do not clearly see any valid and convincing reason why the
4221, provinces were given the option to apply the law by simply President cannot grant executive clemency in administrative cases.
providing for a probation officer. So if a province decides not to It is the courts considered view that if the President can grant
install a probation officer, then the accused within said province will reprieves, commutations and pardons, and remit fines and
be unduly deprived of the provisions of the Probation Law. forfeitures in criminal cases, with much more reason can she grant
Undue Delegation of Legislative Power executive clemency in administrative cases, which are clearly less
There is undue delegation of legislative power. Act 4221 provides serious than criminal offenses.
that it shall only apply to provinces where the respective provincial The court stressed, however, that when we say the President can
boards have provided for a probation officer. But nowhere in the grant executive clemency in administrative cases, we refer only to
law did it state as to what standard (sufficient standard test) should all administrative cases in the Executive branch, not in the Judicial
provincial boards follow in determining whether or not to apply the or Legislative branches of the government.
probation law in their province. This only creates a roving
26
In criminal cases, the quantum of evidence required to convict an sentence.
individual is proof beyond reasonable doubt. On the other hand, in
administrative cases, the quantum of evidence required is mere Issue:
substantial evidence to support a decision. whether or not conviction of a crime by final judgment of a court is
necessary before the petitioner can be validly rearrested and
recommitted for violation of the terms of his conditional pardon
Pardon distinguish from parole and accordingly to serve the balance of his original sentence

TORRES V. GONZALES, G.R. NO. 76872 JULY 23, 1987 Held:


Facts: The status of our case law on the matter under consideration may
Sometime before 1979, petitioner was convicted by the Court of be summed up in the following propositions:
First Instance of Manila of the crime of estafa (two counts) and was
sentenced to an aggregate prison term of from eleven (11) years, 1. The grant of pardon and the determination of the terms and
ten (10) months and twenty-two (22) days to thirty-eight (38) years, conditions of a conditional pardon are purely executive acts which
nine (9) months and one (1) day, and to pay an indemnity of are not subject to judicial scrutiny.
P127,728.75. These convictions were affirmed by the Court of
Appeals. The maximum sentence would expire on 2 November 2. The determination of the occurrence of a breach of a condition
2000. of a pardon, and the proper consequences of such breach, may be
On 18 April 1979, a conditional pardon was granted to the either a purely executive act, not subject to judicial scrutiny under
petitioner by the President of the Philippines on condition that Section 64 (i) of the Revised Administrative Code; or it may be a
petitioner would not again violate any of the penal laws of the judicial act consisting of trial for and conviction of violation of a
Philippines. Should this condition be violated, he will be proceeded conditional pardon under Article 159 of the Revised Penal Code.
against in the manner prescribed by law. Petitioner accepted the Where the President opts to proceed under Section 64 (i) of the
conditional pardon and was consequently released from Revised Administrative Code, no judicial pronouncement of guilt of
confinement. a subsequent crime is necessary, much less conviction therefor by
On 21 May 1986, the Board of Pardons and Parole (the Board) final judgment of a court, in order that a convict may be
resolved to recommend to the President the cancellation of the recommended for the violation of his conditional pardon.
conditional pardon granted to the petitioner. The evidence before
the Board showed that on 22 March 1982 and 24 June 1982, 3. Because due process is not semper et unique judicial process,
petitioner had been charged with twenty counts of estafa in and because the conditionally pardoned convict had already been
Criminal Cases Nos. Q-19672 and Q-20756, which cases were then accorded judicial due process in his trial and conviction for the
(on 21 May 1986) pending trial before the Regional Trial Court of offense for which he was conditionally pardoned, Section 64 (i) of
Rizal (Quezon City). The record before the Board also showed that the Revised Administrative Code is not afflicted with a constitutional
on 26 June 1985, petitioner had been convicted by the Regional vice.
Trial Court of Rizal (Quezon City) of the crime of sedition in Criminal
Case No. Q-22926; this conviction was then pending appeal before A convict granted conditional pardon, like the petitioner herein,
the Intermediate Appellate Court. The Board also had before it a who is recommitted must of course be convicted by final judgment
letter report dated 14 January 1986 from the National Bureau of of a court of the subsequent crime or crimes with which he was
Investigation (NBI), addressed to the Board, on the petitioner. Per charged before the criminal penalty for such subsequent offense(s)
this letter, the records of the NBI showed that a long list of charges can be imposed upon him. Again, since Article 159 of the Revised
had been brought against the petitioner during the last twenty Penal Code defines a distinct, substantive, felony, the parolee or
years for a wide assortment of crimes including estafa, other forms convict who is regarded as having violated the provisions thereof
of swindling, grave threats, grave coercion, illegal possession of must be charged, prosecuted and convicted by final judgment
firearms, ammunition and explosives, malicious mischief, violation before he can be made to suffer the penalty prescribed in Article
of Batas Pambansa Blg. 22, and violation of Presidential Decree No. 159.
772 (interfering with police functions). Some of these charges were
identified in the NBI report as having been dismissed. The NBI Succinctly put, in proceeding against a convict who has been
report did not purport to be a status report on each of the charges conditionally pardoned and who is alleged to have breached the
there listed and identified. conditions of his pardon, the Executive Department has two options:
On 4 June 1986, the respondent Minister of Justice wrote to the (i) to proceed against him under Section 64 (i) of the Revised
President of the Philippines informing her of the Resolution of the Administrative Code; or (ii) to proceed against him under Article 159
Board recommending cancellation of the conditional pardon of the Revised Penal Code which imposes the penalty of prision
previously granted to petitioner. correccional, minimum period, upon a convict who having been
On 8 September 1986, the President cancelled the conditional granted conditional pardon by the Chief Executive, shall violate any
pardon of the petitioner. of the conditions of such pardon. Here, the President has chosen to
On 10 October 1986, the respondent Minister of Justice issued by proceed against the petitioner under Section 64 (i) of the Revised
authority of the President an Order of Arrest and Recommitment Administrative Code. That choice is an exercise of the Presidents
against petitioner. The petitioner was accordingly arrested and executive prerogative and is not subject to judicial scrutiny.
confined in Muntinlupa to serve the unexpired portion of his
27
through falsification of public documents. She was sentenced to jail
Pardon distinguished from amnesty and to indemnify the government in the sum of P4,892.50.The SC
affirmed the decision. She then filed a motion for reconsideration
Norberto Jimenez & Loreto Barrioquinto vs Fernandez but while said motion was pending, she was extended by then
G.R. No. L-1278, January 21, 1949 President Marcos absolute pardon which she accepted (at that time,
the rule was that clemency could be given even before conviction).
Amnesty Compared w/ Pardon Admission Not Needed in Amnesty By reason of said pardon, petitioner wrote the Calbayog City
FACTS treasurer requesting that she be restored to her former post as
Jimenez and Barrioquinto were charged for murder for the assistant city treasurer since the same was still vacant. Her letter
killings they made during the war. The case was proceeded against was referred to the Minister of Finance who ruled that she may be
Jimenez because Barrioquinto was nowhere to be found. Jimenez reinstated to her position without the necessity of a new
was then sentenced to life imprisonment. appointment not earlier than the date she was extended the
Before the period for perfecting an appeal had expired, the absolute pardon.
defendant Jimenez became aware of Proclamation No. 8, which
grants amnesty in favor of all persons who may be charged with an Petitioner wrote the Ministry stressing that the full pardon
act penalized under the RPC in furtherance of the resistance to the bestowed on her has wiped out the crime which implies that her
enemy or against persons aiding in the war efforts of the enemy. service in the government has never been interrupted and
Barrioquinto learned about the proclamation and he surfaced therefore the date of her reinstatement should correspond to the
in order to invoke amnesty as well. However, Commissioner date of her preventive suspension; that she is entitled to backpay
Fernandez of the 14th Amnesty Commission refused to process the for the entire period of her suspension; and that she should not be
amnesty request of the two accused because the two refused to required to pay the proportionate share of the amount of P4,892.50
admit to the crime as charged. Jimenez & Barrioquinto in fact said
that a certain Tolentino was the one who committed the crime The Ministry referred the issue to the Office of the President.
being charged to them. Deputy Executive Secretary Factoran denied Monsantos request
averring that Monsanto must first seek appointment and that the
ISSUE: pardon does not reinstate her former position.
Whether or not admission of guilt is necessary in amnesty.
Issues:
HELD: 1. Is a public officer, who has been granted an absolute pardon by
Pardon is granted by the President and as such it is a private the Chief Executive, entitled to reinstatement to her former
act which must be pleaded and proved by the person pardoned, position without need of a new appointment?
because the courts take no notice thereof; while amnesty by 2. May petitioner be exempt from the payment of the civil
Proclamation of the President with the concurrence of Congress, indemnity imposed upon her by the sentence?
and it is a public act of which the courts should take judicial notice.
Pardon is granted to one after conviction; while amnesty is Held:
granted to classes of persons or communities who may be guilty of The pardon granted to petitioner has resulted in removing her
political offenses, generally before or after the institution of the disqualification from holding public employment but it cannot go
criminal prosecution and sometimes after conviction. beyond that. To regain her former post as assistant city treasurer,
Pardon looks forward and relieves the offender from the she must re-apply and undergo the usual procedure required for a
consequences of an offense of which he has been convicted, that is, new appointment. Civil liability arising from crime is governed by
it abolishes or forgives the punishment, and for that reason it does the Revised Penal Code. It subsists notwithstanding service of
nor work the restoration of the rights to hold public office, or the sentence, or for any reason the sentence is not served by pardon,
right of suffrage, unless such rights be expressly restored by the amnesty or commutation of sentence. Petitioner's civil liability may
terms of the pardon, and it in no case exempts the culprit from only be extinguished by the same causes recognized in the Civil
the payment of the civil indemnity imposed upon him by the Code, namely: payment, loss of the thing due, remission of the debt,
sentence (art 36, RPC). While amnesty looks backward and merger of the rights of creditor and debtor, compensation and
abolishes and puts into oblivion the offense itself, it so overlooks novation.
and obliterates the offense with which he is charged that the person
released by amnesty stands before the law precisely as though he
had committed no offense. Effects of violation of terms of pardon (maybe rearrested by order
of the president; purely executive prerogative under sec. 64 of the
Administrative code)
Effects of pardon (absolute pardon remits penal consequences of
the crime, but not erases the crime itself) Petitioner: WILFREDO TORRES Y SUMULONG
Respondents: HON. NEPTALI A. GONZALES, THE CHAIRMAN,
Monsanto vs. Factoran BOARD OF PARDONS AND PAROLE, and THE DIRECTOR, BUREAU
Facts: OF PRISONS
The Sandiganbayan convicted petitioner Salvacion A. Monsanto FACTS:
(then assistant treasurer of Calbayog City) of the crime of estafa 1978, Torres was convicted of estafa. In 1979, he was pardoned by
28
the president w/ the condition that he shall not violate any penal Petion was denied.
laws again. Should this condition be violated, he will be proceeded
against in the manner prescribed by law. Petitioner accepted the According to the petitioner they were given conditional amnesty on
conditional pardon and was consequently released from Feb. 2, 1985, through the 3rd and 11th amnesty commission of Lanao
confinement. In 1982, Torres was charged with multiple crimes of del Sur and Marawi city subject to the approval or final action of the
estafa. In 1986, Gonzales petitioned for the cancellation of Torres President pursuant to PD 1082.
pardon. Hence, the president cancelled the pardon. Torres appealed
the issue before the SC averring that the Exec Dept erred in The amnesty commission recommended the approval of their
convicting him for violating the conditions of his pardon because amnesty or grant of executive clemency.
the estafa charges against him were not yet final and executory as
they were still on appeal. January 22, 1986, former governor made written representations
with Marcos concerning the applications during the political rally.
ISSUE: Marcos wrote the following on the upper write hand corner of
whether or not conviction of a crime by final judgment of a court is Dimaporos letter: Approved and signed the same with a partly
necessary before the petitioner can be validly rearrested and illegible date.
recommitted for violation of the terms of his conditional pardon
and accordingly to serve the balance of his original sentence. Petitioners also state that the original copies were lost or destroyed
at Malacanang during the bloodless military revolution and could
HELD: not be located.
In proceeding against a convict who has been conditionally
pardoned and who is alleged to have breached the conditions of his Issue:
pardon, the Executive Department has two options: (1) Section 64 (i) Won the PD 1082 applies to the petitioners
of the Revised Administrative Code, a purely executive act, not
subject to judicial scrutiny, or (2) Article 159 of the Revised Penal Held:
Code, a judicial act consisting of trial for and conviction of violation No
of a conditional pardon. It does not apply to them because their acts are nor offenses of
Where the President opts to proceed under Section 64 (i) of the rebellion.
Revised Administrative Code, no judicial pronouncement of guilt of The offenders of which amnesty may be granted under the
a subsequent crime is necessary, much less conviction therefor by provisions pf PD 1082 are acts which were done in furtherance of
final judgment of a court, in order that a convict may be resistance to the republic bay members and supporters of the MILF
recommended for the violation of his conditional pardon. and the Bangsa moro Army and other anti-government group with
Under art. 159 of the RPC, parolee or convict who is regarded as similar motives and aims.
having violated the provisions thereof must be charged, prosecuted Petitioners fall under persons expressly disqualified from amnesty
and convicted by final judgment before he can be made to suffer stated on sec 2(a) of PD 1182, which repealed PD 1082. Petitions
the penalty prescribed. also filed beyond the time limit provided.
In the case at bar, President has chosen to proceed against the The supposed approval of the former president on 1985, in clear
petitioner under Section 64 (i) of the Revised Administrative Code. and conflict with the restrictions in the very decree he promulgated,
That choice is an exercise of the Presidents executive prerogative cannot be given any legal effect. Marcos did not amend his own
and is not subject to judicial scrutiny. decree and he must be held to the terms and conditions that he
*Who determines if violated? The PRESIDENT. When the person was himself had promulgated in the exercise of legislative power.
conditionally pardoned it was a generous exercise by the Chief
Executive of his constitutional prerogative. The acceptance thereof
by the convict or prisoner carrie[d] with it the authority or power of Military Powers (includes:
the Executive to determine whether a condition or conditions of the
pardon has or have been violated. To no other department of the 1. Calling out
Government [has] such power been intrusted.
IBP vs. Zamora
G.R. No.141284, August 15, 2000
TOPIC: Who may avail amnesty Facts:
Invoking his powers as Commander-in-Chief under Sec. 18, Art. VII
MACAGA-AN VS PEOPLE JULY 29, 1987 of the Constitution, the President directed the AFP Chief of Staff and
In a decision on July 15, 1981 the 22 petitioners were charged and PNP Chief to coordinate with each other for the proper deployment
convicted in 33 cases for estafa through falsification of public and and utilization of the Marines to assist the PNP in preventing or
commercial documents) The total amount of government funds suppressing criminal or lawless violence. The President declared
(treasury warrants) involved over P2.7 M that the services of the Marines in the anti-crime campaign are
Petitioners moved to close their cases on ground that they had been merely temporary in nature and for a reasonable period only, until
given amnesty by former President Marcos on January 28 1986. The such time when the situation shall have improved. The IBP filed a
Sandigan Bayan required them to submit originals or authenticated petition seeking to declare the deployment of the Philippine
copies of their amnesty paper, which they were unable to produce. Marines null and void and unconstitutional.
29
destroy the civilian character of the PNP.
Issues:
(1) Whether or not the Presidents factual determination of the
necessity of calling the armed forces is subject to judicial review 2. calling out power, declaration of state of rebellion
(2) Whether or not the calling of the armed forces to assist the PNP G.R. No. 147780. May 10, 2001.]
in joint visibility patrols violates the constitutional provisions on PANFILO LACSON, MICHAEL RAY B. AQUINO and CESAR O.
civilian supremacy over the military and the civilian character of the MANCAO, petitioners v. SECRETARYHERNANDO PEREZ,
PNP P/DIRECTOR LEANDRO MENDOZA, and P/SR. SUPT.
REYNALDOBERROYA,respondents
Held: FACTS:
When the President calls the armed forces to prevent or suppress On May 1, 2001, President Macapagal-Arroyo, faced by an armed
lawless violence, invasion or rebellion, he necessarily exercises a mob assaulting and attempting to break into Malacaang, issued
discretionary power solely vested in his wisdom. Under Sec. 18, Art. Proclamation No. 38 declaring that there was a state of rebellion in
VII of the Constitution, Congress may revoke such proclamation of NCR. She also issued General Order No. 1 directing the AFP and the
martial law or suspension of the privilege of the writ of habeas PNP to suppress the rebellion. Warrantless arrests of several alleged
corpus and the Court may review the sufficiency of the factual basis leaders and promoters of the "rebellion" followed. Aggrieved, 4
thereof. However, there is no such equivalent provision dealing with related petitions were filed before the Court. The case at bar is for
the revocation or review of the Presidents action to call out the prohibition, injunction, mandamus, and habeas corpus (with an
armed forces. The distinction places the calling out power in a urgent application for the issuance of temporary restraining order
different category from the power to declare martial law and power and/or writ of preliminary injunction). Petitioners assail the
to suspend the privilege of the writ of habeas corpus, otherwise, declaration of a state of rebellion by PGMA and the warrantless
the framers of the Constitution would have simply lumped together arrests allegedly effected by virtue thereof, as having no basis both
the 3 powers and provided for their revocation and review without in fact and in law. On May 6, 2001, PGMA ordered the lifting of the
any qualification. declaration of a "state of rebellion" in Metro Manila. Accordingly,
the instant petitions have been rendered moot and academic. As to
The reason for the difference in the treatment of the said powers petitioners' claim that the proclamation of a "state of rebellion" is
highlights the intent to grant the President the widest leeway and being used by the authorities to justify warrantless arrests, the
broadest discretion in using the power to call out because it is Secretary of Justice denies that it has issued a particular order to
considered as the lesser and more benign power compared to the arrest specific persons in connection with the "rebellion."
power to suspend the privilege of the writ of habeas corpus and the
power to impose martial law, both of which involve the curtailment ISSUE:
and suppression of certain basic civil rights and individual freedoms, Whether or not there is a valid warrantless arrest against the
and thus necessitating safeguards by Congress and review by the petitioners.
Court.
HELD:
In view of the constitutional intent to give the President full No. In quelling or suppressing the rebellion, the authorities may
discretionary power to determine the necessity of calling out the only resort to warrantless arrests of persons suspected of rebellion,
armed forces, it is incumbent upon the petitioner to show that the as provided under Section 5, Rule 113 of the Rules of Court, if the
Presidents decision is totally bereft of factual basis. The present circumstances so warrant. The warrantless arrest feared by
petition fails to discharge such heavy burden, as there is no petitioners is, thus, not based on the declaration of a "state of
evidence to support the assertion that there exists no justification rebellion." Petitioners' contention that they are under imminent
for calling out the armed forces. danger of being arrested without warrant do not justify their resort
to the extraordinary remedies of mandamus and prohibition, since
The Court disagrees to the contention that by the deployment of an individual subjected to warrantless arrest is not without
the Marines, the civilian task of law enforcement is militarized in adequate remedies in the ordinary course of law. The prayer for
violation of Sec. 3, Art. II of the Constitution. The deployment of the prohibition and Mandamus is improper at this time. As regards
Marines does not constitute a breach of the civilian supremacy petitioners' prayer that the hold departure orders issued against
clause. The calling of the Marines constitutes permissible use of them be declared null and void ab initio, it is to be noted that
military assets for civilian law enforcement. The local police forces petitioners are not directly assailing the validity of the subject hold
are the ones in charge of the visibility patrols at all times, the real departure orders in their petition. They are not even expressing
authority belonging to the PNP intention to leave the country in the near future. The prayer to set
aside the same must be made in proper proceedings initiated for
Moreover, the deployment of the Marines to assist the PNP does that purpose. Anent petitioners' allegations ex abundante ad
not unmake the civilian character of the police force. The real cautelam in support of their application for the issuance of a writ of
authority in the operations is lodged with the head of a civilian habeas corpus, it is manifest that the writ is not called for since its
institution, the PNP, and not with the military. Since none of the purpose is to relieve petitioners from unlawful restraint, a matter
Marines was incorporated or enlisted as members of the PNP, there which remains speculative up to this very day. Petition is DISMISSED.
can be no appointment to civilian position to speak of. Hence, the However, respondents, consistent and congruent with their
deployment of the Marines in the joint visibility patrols does not undertaking earlier adverted to, together with their agents,
30
representatives, and all persons acting for and in their behalf, are 18, Artilce VII do hereby command the Armed Forces of the
hereby enjoined from arresting petitioners therein without the Philippines, to maintain law and order throughout the Philippines,
required judicial warrant forall acts committed in relation to or in prevent or suppress all forms of lawless violence as well any act of
connection with the May 1, 2001 siege of Malacaang. insurrection or rebellion
Second provision: and to enforce obedience to all the laws
and to all decrees, orders and regulations promulgated by me
As distinguished in the Marcos time personally or upon my direction;
Third provision: as provided in Section 17, Article XII of the
DAVID VS MACAPAGAL - ARROYO Constitution do hereby declare a State of National Emergency.
G.R. No. 171396, May 3 2006 [Legislative Department - Power to
Declare War and Delegate Emergency Power] PP 1017 is partially constitutional insofar as provided by the first
FACTS: provision of the decree.
On February 24, 2006, President Arroyo issued PP No. 1017 First Provision: Calling Out Power.
declaring a state of emergency, thus: The only criterion for the exercise of the calling-out power is that
whenever it becomes necessary, the President may call the armed
NOW, THEREFORE, I, Gloria Macapagal-Arroyo, President of the forces to prevent or suppress lawless violence, invasion or
Republic of the Philippines and Commander-in-Chief of the Armed rebellion. (Integrated Bar of the Philippines v. Zamora)
Forces of the Philippines, [calling-out power] by virtue of the President Arroyos declaration of a state of rebellion was merely
powers vested upon me by Section 18, Article 7 of the Philippine an act declaring a status or condition of public moment or interest,
Constitution which states that: The President. . . whenever it a declaration allowed under Section 4, Chap 2, Bk II of the Revised
becomes necessary, . . . may call out (the) armed forces to prevent Administration Code. Such declaration, in the words of Sanlakas, is
or suppress. . .rebellion. . ., and in my capacity as their harmless, without legal significance, and deemed not written. In
Commander-in-Chief, do hereby command the Armed Forces of the these cases, PP 1017 is more than that. In declaring a state of
Philippines, to maintain law and order throughout the Philippines, national emergency, President Arroyo did not only rely on Section
prevent or suppress all forms of lawless violence as well as any act 18, Article VII of the Constitution, a provision calling on the AFP to
of insurrection or rebellion ["take care" power] and to enforce prevent or suppress lawless violence, invasion or rebellion. She
obedience to all the laws and to all decrees, orders and regulations also relied on Section 17, Article XII, a provision on the States
promulgated by me personally or upon my direction; and [power to extraordinary power to take over privately-owned public utility and
take over] as provided in Section 17, Article 12 of the Constitution business affected with public interest. Indeed, PP 1017 calls for
do hereby declare a State of National Emergency. the exercise of an awesome power. Obviously, such Proclamation
cannot be deemed harmless.
On the same day, PGMA issued G.O. No. 5 implementing PP1017, To clarify, PP 1017 is not a declaration of Martial Law. It is merely
directing the members of the AFP and PNP "to immediately carry an exercise of President Arroyos calling-out power for the armed
out the necessary and appropriate actions and measures to forces to assist her in preventing or suppressing lawless violence.
suppress and prevent acts of terrorism and lawless violence."
Second Provision: The "Take Care" Power.
David, et al. assailed PP 1017 on the grounds that (1) it encroaches The second provision pertains to the power of the President to
on the emergency powers of Congress; (2) it is a subterfuge to avoid ensure that the laws be faithfully executed. This is based on
the constitutional requirements for the imposition of martial law; Section 17, Article VII which reads:
and (3) it violates the constitutional guarantees of freedom of the SEC. 17. The President shall have control of all the executive
press, of speech and of assembly. They alleged direct injury departments, bureaus, and offices. He shall ensure that the laws be
resulting from illegal arrest and unlawful search committed by faithfully executed.
police operatives pursuant to PP 1017. This Court rules that the assailed PP 1017 is unconstitutional insofar
as it grants President Arroyo the authority to promulgate decrees.
During the hearing, the Solicitor General argued that the issuance of Legislative power is peculiarly within the province of the Legislature.
PP 1017 and GO 5 have factual basis, and contended that the intent Section 1, Article VI categorically states that [t]he legislative power
of the Constitution is to give full discretionary powers to the shall be vested in the Congress of the Philippines which shall consist
President in determining the necessity of calling out the armed of a Senate and a House of Representatives. To be sure, neither
forces. The petitioners did not contend the facts stated b the Martial Law nor a state of rebellion nor a state of emergency can
Solicitor General. justify President Arroyos exercise of legislative power by issuing
decrees.
ISSUE:
Whether or not the PP 1017 and G.O. No. 5 is constitutional. Third Provision: The Power to Take Over
Distinction must be drawn between the Presidents authority to
RULING: declare a state of national emergency and to exercise emergency
The operative portion of PP 1017 may be divided into three powers. To the first, Section 18, Article VII grants the President
important provisions, thus: such power, hence, no legitimate constitutional objection can be
raised. But to the second, manifold constitutional issues arise.
First provision: by virtue of the power vested upon me by Section Generally, Congress is the repository of emergency powers. This is
31
evident in the tenor of Section 23 (2), Article VI authorizing it to charge sheet but only the subversion and murder cases against
delegate such powers to the President. Certainly, a body cannot Buscayno were decided by the Military Commission with the
delegate a power not reposed upon it. However, knowing that penalty of death by firing squad although the decision is still subject
during grave emergencies, it may not be possible or practicable for to review.
Congress to meet and exercise its powers, the Framers of our Bernabe Buscayno alias Commander Dante and Jose Ma. Sison
Constitution deemed it wise to allow Congress to grant emergency alias Amado Guerrero, alleged subversives classified as
powers to the President, subject to certain conditions, thus: "PKP/HMB/CPP/MAMAO and Traditional Armed Group
(1) There must be a war or other emergency. personalities," were wanted by the authorities since 1971.
(2) The delegation must be for a limited period only. Buscayno and Sison were included in the so-called "National
(3) The delegation must be subject to such restrictions as the Target List" of active participants in the conspiracy to seize political
Congress may prescribe. and state power and to take over the government by force whose
(4) The emergency powers must be exercised to carry out a arrest was ordered under General Order No. 2 dated September 22,
national policy declared by Congress. 1972. The list was prepared by Colonel Hamilton B. Dimaya. (p. 95,
Section 17, Article XII must be understood as an aspect of the Rollo of L-47185)
emergency powers clause. The taking over of private business On October 2, 1981, Buscayno and the Sison spouses filed the
affected with public interest is just another facet of the emergency instant omnibus, catchall petition for habeas corpus, prohibition
powers generally reposed upon Congress. Thus, when Section 17 and mandamus couched in repetitious, involuted and obfuscatory
states that the the State may, during the emergency and under verbiage.
reasonable terms prescribed by it, temporarily take over or direct They prayed that the decision of Military Commission No. 2
the operation of any privately owned public utility or business dated May 4, 1981, convicting Buscayno of subversion and murder
affected with public interest, it refers to Congress, not the and sentencing him to death by firing squad, be declared void
President. Now, whether or not the President may exercise such because he was denied his constitutional right to present evidence
power is dependent on whether Congress may delegate it to him and that he be released from detention.
pursuant to a law prescribing the reasonable terms thereof. They also prayed that the charges of rebellion and subversion
Following our interpretation of Section 17, Article XII, invoked by be dismissed for being in contravention of the rule on double
President Arroyo in issuing PP 1017, this Court rules that such jeopardy, that Military Commissions Nos. 1, 6 and 25 be enjoined
Proclamation does not authorize her during the emergency to from proceeding with the trial of the petitioners and that the
temporarily take over or direct the operation of any privately owned petitioners be released. They also prayed that they be granted bail.
public utility or business affected with public interest without The petitioners also asked for the issuance of a temporary
authority from Congress. restraining order, enjoining the three Commissions from trying the
Let it be emphasized that while the President alone can declare a petitioners, enjoining Military Commission No. 1 from continuing
state of national emergency, however, without legislation, he has no with the perpetuation of testimonies and from requiring the
power to take over privately-owned public utility or business petitioners to attend the perpetuation proceedings and enjoining
affected with public interest. Nor can he determine when such the Review Board-AFP from reviewing the decision in the
exceptional circumstances have ceased. Likewise, without subversion and murder cases.
legislation, the President has no power to point out the types of Petitioner contends that criminal liability for subversion was
businesses affected with public interest that should be taken over. extinguished when Presidential Decree No. 885 (which took effect
In short, the President has no absolute authority to exercise all the on May 11, 1976, 72 OG 3826) repealed Republic Act No. 1700.
powers of the State under Section 17, Article VII in the absence of
an emergency powers act passed by Congress. ISSUE:
Whether or not, criminal liability for subversion be
As of G.O. No. 5, it is constitutional since it provides a standard by extinguished for acts committed prior to the effectivity of
which the AFP and the PNP should implement PP 1017, i.e. Presidential Decree No. 885 amending Republic Act No. 1700, the
whatever is necessary and appropriate actions and measures to Anti-Subversion Law.
suppress and prevent acts of lawless violence. Considering that RULING:
acts of terrorism have not yet been defined and made punishable Petition dismissed. The acts committed before the effectivity of
by the Legislature, such portion of G.O. No. 5 is declared Presidential Decree No. 885 shall be prosecuted and punished
unconstitutional. under Republic Act No. 1700.
Revised Anti-Subversion Law (P.D. No. 885), in repealing or
superseding Republic Act No. 1700, expressly provides in its Section
BUSCAYNO VS MILITARY COMMISSION, 7 that "acts committed in violation'' of the former law before the
G.R. NO. 58284, 19 NOVEMBER 1981 effectivity of said decree "shall be prosecuted and punished in
FACTS: accordance with the provisions of the former Act" and that nothing
Petitioners Bernabe Buscayno Jose Ma. Sison And Juliet Sison, in said decree "shall prevent prosecution of cases pending for
were charged with subversion by the Military Commission in violation of" Republic Act No. 1700. That saving or transitory clause
different charge sheets on different dates for acts committed prior is re-enacted in Section 14(i) of the National Security Code.
to the effectivity of Presidential Decree No. 885 on May 11, 1976
amending Republic Act No. 1700, the Anti-Subversion Law. The NOTES:
three petitioners were all charged with rebellion in an amended REPUBLIC ACT NO. 1700, ALSO KNOWN AS THE
32
ANTI-SUBVERSION ACT, is a law that outlaws the Communist Party the joint houses of Congress could fulfill their automatic duty to
of the Philippines (CPP) and similar organizations, penalizing mere review and validate or invalidate the same. xxx.
membership of these organizations. It declares that the Communist xxx xxx
Party of the Philippines, although purportedly a political party, is in [U]nder the 1987 Constitution the President and the Congress act in
fact an organized conspiracy to overthrow the Government of the tandem in exercising the power to proclaim martial law or suspend
Republic of the Philippines not only by force and violence but also the privilege of the writ of habeas corpus. They exercise the
by deceit, subversion and other illegal means, for the purpose of power, not only sequentially, but in a sense jointly since, after the
establishing in the Philippines a totalitarian regime subject to alien President has initiated the proclamation or the suspension, only the
domination and control. Under the Anti-Subversion Act, the term Congress can maintain the same based on its own evaluation of the
Communist Party of the Philippines means and includes the situation on the ground, a power that the President does not have.
organizations now known as the Communist Party of the Philippines Consequently, although the Constitution reserves to the Supreme
and its military arm, the Hukbong Mapagpalaya ng Bayan, formerly Court the power to review the sufficiency of the factual basis of the
known as HUKBALAHAPS, and any successors of such organizations. proclamation or suspension in a proper suit, it is implicit that the
PRESIDENTIAL DECREE NO. 885: OUTLAWING SUBVERSIVE Court must allow Congress to exercise its own review powers, which
ORGANIZATIONS, PENALIZING MEMBERSHIP THEREIN AND FOR is automatic rather than initiated. Only when Congress defaults in
OTHER PURPOSES. its express duty to defend the Constitution through such review
should the Supreme Court step in as its final rampart. The
constitutional validity of the Presidents proclamation of martial law
Philip Sigfrid Fortun v. Gloria Macapagal-Arroyo, et al., G.R. No. or suspension of the writ of habeas corpus is first a political
190293, March 20, 2012 (and other consolidated cases) question in the hands of Congress before it becomes a justiciable
I. THE FACTS one in the hands of the Court.
On November 23, 2009, heavily armed men believed led by the xxx xxx
ruling Ampatuan family of Maguindanao gunned down and buried Here, President Arroyo withdrew Proclamation 1959 before the
under shoveled dirt 57 innocent civilians. In response to this joint houses of Congress, which had in fact convened, could act on
carnage, President Arroyo issued on November 24, 2009 PP 1946 the same. Consequently, the petitions in these cases have become
declaring a state of emergency in Maguindanao, Sultan Kudarat, and moot and the Court has nothing to review. The lifting of martial
Cotabato City. law and restoration of the privilege of the writ of habeas corpus in
On December 4, 2009, President Arroyo issued PP 1959 declaring Maguindanao was a supervening event that obliterated any
martial law and suspending the privilege of the writ of habeas justiciable controversy.
corpus in Maguindanao except for identified areas of the Moro Two. Since President Arroyo withdrew her proclamation of martial
Islamic Liberation Front. On December 6, 2009, President Arroyo law and suspension of the privilege of the writ of habeas corpus in
submitted her report to Congress. On December 9, 2009, Congress just eight days, they have not been meaningfully implemented.
convened in joint session to review the validity of the Presidents The military did not take over the operation and control of local
action. But two days later, or on December 12, 2009, before government units in Maguindanao. The President did not issue
Congress could act, the President issued PP 1963, lifting martial law any law or decree affecting Maguindanao that should ordinarily be
and restoring the privilege of the writ of habeas corpus. enacted by Congress. No indiscriminate mass arrest had been
reported. Those who were arrested during the period were either
II. THE ISSUES released or promptly charged in court. Indeed, no petition for
Did the issuance of PP 1963, lifting martial law and restoring the habeas corpus had been filed with the Court respecting arrests
[privilege of the] writ in Maguindanao, render the issues moot and made in those eight days. The point is that the President intended
academic? by her action to address an uprising in a relatively small and
sparsely populated province. In her judgment, the rebellion was
III. THE RULING localized and swiftly disintegrated in the face of a determined and
[The Court DISMISSED the consolidated petitions on the ground amply armed government presence.
that they have become MOOT and ACADEMIC.] xxx xxx xxx.
YES, the issuance of PP 1963, lifting martial law and restoring the In a real sense, the proclamation and the suspension never took off.
[privilege of the] writ in Maguindanao, rendered the issues moot The Congress itself adjourned without touching the matter, it having
and academic become moot and academic.
Prudence and respect for the co-equal departments of the
government dictate that the Court should be cautious in
entertaining actions that assail the constitutionality of the acts of Emergency power vs Declaration of Sate of emergency
the Executive or the Legislative department. The issue of
constitutionality, said the Court in Biraogo v. Philippine Truth Ampatuan, et al. v. Secretary Ronaldo Puno
Commission of 2010, must be the very issue of the case, that the G.R. No. 190259 : June 7, 2011
resolution of such issue is unavoidable. DATU ZALDY UY AMPATUAN, ANSARUDDIN ADIONG, REGIE
The issue of the constitutionality of Proclamation 1959 is not SAHALI-GENERALE, Petitioners, v. HON. RONALDO PUNO, in his
unavoidable for two reasons: capacity as Secretary of the Department of Interior and Local
One. President Arroyo withdrew her proclamation of martial law Government and alter-ego of President Gloria Macapagal-Arroyo,
and suspension of the privilege of the writ of habeas corpus before and anyone acting in his stead and on behalf of the President of the
33
Philippines, ARMED FORCES OF THE PHILIPPINES (AFP), or any of the assailed proclamation and administrative orders did not provide
their units operating in the Autonomous Region in Muslim for the exercise of emergency powers.
Mindanao (ARMM), and PHILIPPINE NATIONAL POLICE, or any of ISSUES:
their units operating in ARMM, Respondents. [1] Whether the aforementioned issuances are constitutional
[2] Whether or not President Arroyo invalidly exercised emergency
FACTS: powers when she called out the AFP and the PNP to prevent and
On November 24, 2009, the day after the gruesome massacre of 57 suppress all incidents of lawless violence in Maguindanao, Sultan
men and women, including some news reporters, then President Kudarat, and Cotabato City
Gloria Macapagal-Arroyo issued Proclamation 1946, placing "the
Provinces of Maguindanao and Sultan Kudarat and the City of HELD:
Cotabato under a state of emergency." She directed the AFP and the The AO Nos 273 and 273-A are constitutional.
PNP "to undertake such measures as may be allowed by the POLITICAL LAW emergency powers
Constitution and by law to prevent and suppress all incidents of The deployment is not by itself an exercise of emergency powers as
lawless violence" in the named places. understood under Section 23 (2), Article VI of the Constitution,
which provides:
Three days later or on November 27, President Arroyo also issued
AO 273 "transferring" supervision of the ARMM from the Office of SECTION 23. x x x (2) In times of war or other national emergency,
the President to the DILG. But, due to issues raised over the the Congress may, by law, authorize the President, for a limited
terminology used in AO 273, the President issued Administrative period and subject to such restrictions as it may prescribe, to
Order 273-A (AO 273-A) amending the former, by "delegating" exercise powers necessary and proper to carry out a declared
instead of "transferring" supervision of the ARMM to the DILG. national policy. Unless sooner withdrawn by resolution of the
Congress, such powers shall cease upon the next adjournment
Claiming that the President's issuances encroached on the ARMM's thereof.
autonomy, petitioners Datu Zaldy Uy Ampatuan, Ansaruddin Adiong,
and Regie Sahali-Generale, all ARMM officials, filed this petition for The President did not proclaim a national emergency, only a state of
prohibition under Rule 65. They alleged that the proclamation and emergency in the three places mentioned. And she did not act
the orders empowered the DILG Secretary to take over ARMMs pursuant to any law enacted by Congress that authorized her to
operations and seize the regional government's powers, in violation exercise extraordinary powers. The calling out of the armed forces
of the principle of local autonomy under Republic Act 9054 (also to prevent or suppress lawless violence in such places is a power
known as the Expanded ARMM Act) and the Constitution. The that the Constitution directly vests in the President. She did not
President gave the DILG Secretary the power to exercise, not merely need a congressional authority to exercise the same.
administrative supervision, but control over the ARMM since the
latter could suspend ARMM officials and replace them. The President's call on the armed forces to prevent or suppress
lawless violence springs from the power vested in her under Section
Petitioners alleged that the deployment of troops and the taking 18, Article VII of the Constitution, which provides -
over of the ARMM constitutes an invalid exercise of the Presidents
emergency powers. Hence, petitioners prayed the nullity of SECTION 18. The President shall be the Commander-in-Chief of all
Proclamation 1946 as well as AOs 273 and 273-A and respondents, armed forces of the Philippines and whenever it becomes necessary,
DILG Secretary, the AFP, and the PNP be enjoined from he may call out such armed forces to prevent or suppress lawless
implementing them. violence, invasion or rebellion. x x x

In its comment for the respondents, the (OSG) insisted that the On the other hand, the President, as Commander-in-Chief has a vast
President issued Proclamation 1946, not to deprive the ARMM of its intelligence network to gather information, some of which may be
autonomy, but to restore peace and order in subject places. She classified as highly confidential or affecting the security of the state.
issued the proclamation pursuant to her "calling out" power as In the exercise of the power to call, on-the-spot decisions may be
Commander-in-Chief under the first sentence of Section 18, Article imperatively necessary in emergency situations to avert great loss of
VII of the Constitution. The determination of the need to exercise human lives and mass destruction of property. Indeed, the decision
this power rests solely on her wisdom.10 She must use her to call out the military to prevent or suppress lawless violence must
judgment based on intelligence reports and such best information be done swiftly and decisively if it were to have any effect at all.
as are available to her to call out the armed forces to suppress and
prevent lawless violence wherever and whenever these reared their Here, petitioners failed to show that the declaration of a state of
ugly heads. emergency in the Provinces of Maguindanao, Sultan Kudarat and
Cotabato City, as well as the President's exercise of the "calling out"
On the other hand, the President merely delegated through AOs 273 power had no factual basis. They simply alleged that, since not all
and 273-A her supervisory powers over the ARMM to the DILG areas under the ARMM were placed under a state of emergency, it
Secretary who was her alter ego any way. These orders did not follows that the takeover of the entire ARMM by the DILG Secretary
authorize a take over of the ARMM. They did not give him blanket had no basis too.
authority to suspend or replace ARMM officials. The delegation was
necessary to facilitate the investigation of the mass killings. Further, Considering the fact that the principal victims of the brutal
34
bloodshed are members of the Mangudadatu family and the main G. R. No. 138570 October 10, 2000
perpetrators of the brutal killings are members and followers of the Facts:
Ampatuan family, both the military and police had to prepare for The United States panel met with the Philippine panel to discussed,
and prevent reported retaliatory actions from the Mangudadatu among others, the possible elements of the Visiting Forces
clan and additional offensive measures from the Ampatuan clan. Agreement (VFA). This resulted to a series of conferences and
negotiations which culminated on January 12 and 13, 1998.
In other words, the imminence of violence and anarchy at the time Thereafter, President Fidel Ramos approved the VFA, which was
the President issued Proclamation 1946 was too grave to ignore and respectively signed by Secretary Siazon and United States
she had to act to prevent further bloodshed and hostilities in the Ambassador Thomas Hubbard.
places mentioned. Progress reports also indicated that there was Pres. Joseph Estrada ratified the VFA on October 5, 1998 and on
movement in these places of both high-powered firearms and May 27, 1999, the senate approved it by (2/3) votes.
armed men sympathetic to the two clans. Thus, to pacify the Cause of Action:
peoples fears and stabilize the situation, the President had to take Petitioners, among others, assert that Sec. 25, Art XVIII of the 1987
preventive action. She called out the armed forces to control the constitution is applicable and not Section 21, Article VII.
proliferation of loose firearms and dismantle the armed groups that Following the argument of the petitioner, under they provision cited,
continuously threatened the peace and security in the affected the foreign military bases, troops, or facilities may be allowed in
places. the Philippines unless the following conditions are sufficiently met:
The Petition is dismissed for lack of merit. a) it must be a treaty,
b) it must be duly concurred in by the senate, ratified by a majority
of the votes cast in a national referendum held for that purpose if so
Diplomatic Power required by congress, and
c) recognized as such by the other contracting state.
Nicolas V. Romulo Respondents, on the other hand, argue that Section 21 Article VII is
*This case is consolidated with Salonga vs Daniel Smith & BAYAN vs applicable so that, what is requires for such treaty to be valid and
Gloria Arroyo effective is the concurrence in by at least two-thirds of all the
On the 1st of November 2005, Daniel Smith committed the crime of members of the senate.
rape against Nicole. He was convicted of the said crime and was
ordered by the court to suffer imprisonment. Smith was a US ISSUE:
serviceman convicted of a crime against our penal laws and the Is the VFA governed by the provisions of Section 21, Art VII or of
crime was committed within the countrys jurisdiction. But pursuant Section 25, Article XVIII of the Constitution?
to the VFA, a treaty between the US and Philippines, the US
embassy was granted custody over Smith. Nicole, together with the HELD:
other petitioners appealed before the SC assailing the validity of the Section 25, Article XVIII, which specifically deals with treaties
VFA. Their contention is that the VFA was not ratified by the US involving foreign military bases, troops or facilities should apply in
senate in the same way our senate ratified the VFA. the instant case. To a certain extent and in a limited sense, however,
the provisions of section 21, Article VII will find applicability with
ISSUE: regard to the issue and for the sole purpose of determining the
Is the VFA void and unconstitutional & whether or not it is number of votes required to obtain the valid concurrence of the
self-executing. senate.
The Constitution, makes no distinction between transient and
HELD: permanent. We find nothing in section 25, Article XVIII that
The VFA is a self-executing Agreement because the parties intend its requires foreign troops or facilities to be stationed or placed
provisions to be enforceable, precisely because the VFA is intended permanently in the Philippines.
to carry out obligations and undertakings under the RP-US Mutual It is inconsequential whether the United States treats the VFA only
Defense Treaty. As a matter of fact, the VFA has been implemented as an executive agreement because, under international law, an
and executed, with the US faithfully complying with its obligation to executive agreement is as binding as a treaty.
produce Smith before the court during the trial.
The VFA is covered by implementing legislation inasmuch as it is the
very purpose and intent of the US Congress that executive PIMENTEL v. EXECUTIVE SECRETARY
agreements registered under this Act within 60 days from their Facts:
ratification be immediately implemented. The SC noted that the VFA This is a petition of Senator Aquilino Pimentel and the other parties
is not like other treaties that need implementing legislation such as to ask the Supreme Court to require the Executive Department to
the Vienna Convention. As regards the implementation of the RP-US transmit the Rome Statute which established the International
Mutual Defense Treaty, military aid or assistance has been given Criminal Court for the Senates concurrence in accordance with Sec
under it and this can only be done through implementing legislation. 21, Art VII of the 1987 Constitution.
The VFA itself is another form of implementation of its provisions. It is the theory of the petitioners that ratification of a treaty, under
both domestic law and international law, is a function of the Senate.
Hence, it is the duty of the executive department to transmit the
BAYAN v. ZAMORA signed copy of the Rome Statute to the Senate to allow it to exercise
35
its discretion with respect to ratification of treaties. Moreover, the same is opened for signature. This step is primarily intended as
petitioners submit that the Philippines has a ministerial duty to a means of authenticating the instrument and for the purpose of
ratify the Rome Statute under treaty law and customary symbolizing the good faith of the parties; but, significantly, it does
international law. Petitioners invoke the Vienna Convention on the not indicate the final consent of the state in cases where ratification
Law of Treaties enjoining the states to refrain from acts which would of the treaty is required. The document is ordinarily signed in
defeat the object and purpose of a treaty when they have signed accordance with the alternat, that is, each of the several negotiators
the treaty prior to ratification unless they have made their intention is allowed to sign first on the copy which he will bring home to his
clear not to become parties to the treaty.[5] own state.
The Office of the Solicitor General, commenting for the respondents, Ratification, which is the next step, is the formal act by which a
questioned the standing of the petitioners to file the instant suit. It state confirms and accepts the provisions of a treaty concluded by
also contended that the petition at bar violates the rule on its representatives. The purpose of ratification is to enable the
hierarchy of courts. On the substantive issue raised by petitioners, contracting states to examine the treaty more closely and to give
respondents argue that the executive department has no duty to them an opportunity to refuse to be bound by it should they find it
transmit the Rome Statute to the Senate for concurrence. inimical to their interests. It is for this reason that most treaties are
made subject to the scrutiny and consent of a department of the
Issue: government other than that which negotiated them.
Whether or not the executive department has a ministerial duty to The last step in the treaty-making process is the exchange of the
transmit the Rome Statute (or any treaty) to the Senate for instruments of ratification, which usually also signifies the effectivity
concurrence. of the treaty unless a different date has been agreed upon by the
parties. Where ratification is dispensed with and no effectivity
Ruling: clause is embodied in the treaty, the instrument is deemed effective
The petition was dismissed. The Supreme Court ruled that the the upon its signature.
President, being the head of state, is regarded as the sole organ and Petitioners arguments equate the signing of the treaty by the
authority in external relations and is the countrys sole Philippine representative with ratification. It should be underscored
representative with foreign nations. As the chief architect of foreign that the signing of the treaty and the ratification are two separate
policy, the President acts as the countrys mouthpiece with respect and distinct steps in the treaty-making process. As earlier discussed,
to international affairs. Hence, the President is vested with the the signature is primarily intended as a means of authenticating the
authority to deal with foreign states and governments, extend or instrument and as a symbol of the good faith of the parties. It is
withhold recognition, maintain diplomatic relations, enter into usually performed by the states authorized representative in the
treaties, and otherwise transact the business of foreign relations. In diplomatic mission. Ratification, on the other hand, is the formal act
the realm of treaty-making, the President has the sole authority to by which a state confirms and accepts the provisions of a treaty
negotiate with other states. concluded by its representative.
Nonetheless, while the President has the sole authority to negotiate It should be emphasized that under our Constitution, the power to
and enter into treaties, the Constitution provides a limitation to his ratify is vested in the President, subject to the concurrence of the
power by requiring the concurrence of 2/3 of all the members of the Senate. The role of the Senate, however, is limited only to giving or
Senate for the validity of the treaty entered into by him. Section 21, withholding its consent, or concurrence, to the ratification. Hence,
Article VII of the 1987 Constitution provides that no treaty or it is within the authority of the President to refuse to submit a
international agreement shall be valid and effective unless treaty to the Senate or, having secured its consent for its ratification,
concurred in by at least two-thirds of all the Members of the refuse to ratify it. Although the refusal of a state to ratify a treaty
Senate. which has been signed in its behalf is a serious step that should not
Justice Isagani Cruz, in his book on International Law, describes the be taken lightly, such decision is within the competence of the
treaty-making process in this wise: President alone, which cannot be encroached by this Court via a
The usual steps in the treaty-making process are: negotiation, writ of mandamus. This Court has no jurisdiction over actions
signature, ratification, and exchange of the instruments of seeking to enjoin the President in the performance of his official
ratification. The treaty may then be submitted for registration and duties.
publication under the U.N. Charter, although this step is not
essential to the validity of the agreement as between the parties.
Negotiation may be undertaken directly by the head of state but he Vinuya v. Romulo
now usually assigns this task to his authorized representatives. FACTS:
These representatives are provided with credentials known as full This is an original Petition for Certiorari under Rule 65 of the Rules
powers, which they exhibit to the other negotiators at the start of of Court with an application for the issuance of a writ of preliminary
the formal discussions. It is standard practice for one of the parties mandatory injunction against the Office of the Executive Secretary,
to submit a draft of the proposed treaty which, together with the the Secretary of the DFA, the Secretary of the DOJ, and the OSG.
counter-proposals, becomes the basis of the subsequent Petitioners are all members of the MALAYA LOLAS, a non-stock,
negotiations. The negotiations may be brief or protracted, non-profit organization registered with the SEC, established for the
depending on the issues involved, and may even collapse in case purpose of providing aid to the victims of rape by Japanese military
the parties are unable to come to an agreement on the points under forces in the Philippines during the Second World War.
consideration. Petitioners claim that since 1998, they have approached the
If and when the negotiators finally decide on the terms of the treaty, Executive Department through the DOJ, DFA, and OSG, requesting
36
assistance in filing a claim against the Japanese officials and military wisdom of such decision is not for the courts to question.
officers who ordered the establishment of the comfort women The President, not Congress, has the better opportunity of knowing
stations in the Philippines. But officials of the Executive Department the conditions which prevail in foreign countries, and especially is
declined to assist the petitioners, and took the position that the this true in time of war. He has his confidential sources of
individual claims of the comfort women for compensation had information. He has his agents in the form of diplomatic, consular
already been fully satisfied by Japans compliance with the Peace and other officials.
Treaty between the Philippines and Japan. The Executive Department has determined that taking up
Hence, this petition where petitioners pray for this court to (a) petitioners cause would be inimical to our countrys foreign policy
declare that respondents committed grave abuse of discretion interests, and could disrupt our relations with Japan, thereby
amounting to lack or excess of discretion in refusing to espouse creating serious implications for stability in this region. For the to
their claims for the crimes against humanity and war crimes overturn the Executive Departments determination would mean an
committed against them; and (b) compel the respondents to assessment of the foreign policy judgments by a coordinate political
espouse their claims for official apology and other forms of branch to which authority to make that judgment has been
reparations against Japan before the International Court of Justice constitutionally committed.
(ICJ) and other international tribunals. From a municipal law perspective, certiorari will not lie. As a general
Respondents maintain that all claims of the Philippines and its principle, where such an extraordinary length of time has lapsed
nationals relative to the war were dealt with in the San Francisco between the treatys conclusion and our consideration the
Peace Treaty of 1951 and the bilateral Reparations Agreement of Executive must be given ample discretion to assess the foreign
1956. policy considerations of espousing a claim against Japan, from the
On January 15, 1997, the Asian Womens Fund and the Philippine standpoint of both the interests of the petitioners and those of the
government signed a Memorandum of Understanding for medical Republic, and decide on that basis if apologies are sufficient, and
and welfare support programs for former comfort women. Over the whether further steps are appropriate or necessary.
next five years, these were implemented by the Department of In the international sphere, traditionally, the only means available
Social Welfare and Development. for individuals to bring a claim within the international legal system
has been when the individual is able to persuade a government to
ISSUE: bring a claim on the individuals behalf. By taking up the case of one
WON the Executive Department committed grave abuse of of its subjects and by resorting to diplomatic action or international
discretion in not espousing petitioners claims for official apology judicial proceedings on his behalf, a State is in reality asserting its
and other forms of reparations against Japan. own right to ensure, in the person of its subjects, respect for the
rules of international law.
RULING: Within the limits prescribed by international law, a State may
Petition lacks merit. From a Domestic Law Perspective, the exercise diplomatic protection by whatever means and to whatever
Executive Department has the exclusive prerogative to determine extent it thinks fit, for it is its own right that the State is asserting.
whether to espouse petitioners claims against Japan. Should the natural or legal person on whose behalf it is acting
Political questions refer to those questions which, under the consider that their rights are not adequately protected, they have
Constitution, are to be decided by the people in their sovereign no remedy in international law. All they can do is resort to national
capacity, or in regard to which full discretionary authority has been law, if means are available, with a view to furthering their cause or
delegated to the legislative or executive branch of the government. obtaining redress. All these questions remain within the province of
It is concerned with issues dependent upon the wisdom, not legality municipal law and do not affect the position internationally.
of a particular measure. Even the invocation of jus cogens norms and erga omnes obligations
One type of case of political questions involves questions of foreign will not alter this analysis. Petitioners have not shown that the
relations. It is well-established that the conduct of the foreign crimes committed by the Japanese army violated jus cogens
relations of our government is committed by the Constitution to the prohibitions at the time the Treaty of Peace was signed, or that the
executive and legislativethe politicaldepartments of the duty to prosecute perpetrators of international crimes is an erga
government, and the propriety of what may be done in the exercise omnes obligation or has attained the status of jus cogens.
of this political power is not subject to judicial inquiry or decision. The term erga omnes (Latin: in relation to everyone) in international
are delicate, complex, and involve large elements of prophecy. They law has been used as a legal term describing obligations owed by
are and should be undertaken only by those directly responsible to States towards the community of states as a whole. Essential
the people whose welfare they advance or imperil. distinction should be drawn between the obligations of a State
But not all cases implicating foreign relations present political towards the international community as a whole, and those arising
questions, and courts certainly possess the authority to construe or vis--vis another State in the field of diplomatic protection. By their
invalidate treaties and executive agreements. However, the very nature, the former are the concern of all States. In view of the
question whether the Philippine government should espouse claims importance of the rights involved, all States can be held to have a
of its nationals against a foreign government is a foreign relations legal interest in their protection; they are obligations erga omnes.
matter, the authority for which is demonstrably committed by our The term jus cogens (literally, compelling law) refers to norms
Constitution not to the courts but to the political branches. In this that command peremptory authority, superseding conflicting
case, the Executive Department has already decided that it is to the treaties and custom. Jus cogens norms are considered peremptory
best interest of the country to waive all claims of its nationals for in the sense that they are mandatory, do not admit derogation, and
reparations against Japan in the Treaty of Peace of 1951. The can be modified only by general international norms of equivalent
37
authority the Chief Executive full discretion to determine whether an alien's
WHEREFORE, the Petition is hereby DISMISSED. residence in the country is so undesirable as to affect or injure the
security welfare or interest of the state.
o The Chief Executive is the sole and exclusive judge of the
Deportation of Undesirable Aliens (essentially an Executive Power) existence of facts which warrant the deportation of aliens as
Go Tek vs. Deportation Board | Aquino disclosed in an investigation conducted in accordance with Sec. 69
FACTS of the RAC.
On March 3, 1964 the chief prosecutor of the Deportation o After all, the inherent right of a country to expel or deport
Board filed a complaint against Go Tek, a chinaman, praying that the aliens because their continued presence is rental to public welfare is
board recommend his immediate deportation to the President absolute and unqualified.
because he was an undesirable alien on the basis of these As the President is granted full discretion as regards
allegations: deportation, it is fundamental that an executive order for
o Go Tek was a sector commander and intelligence and record deportation is not dependent on a prior judicial conviction in a case.
officer of a guerilla unit of the Emergency Intelligence Section,
Army of the United States;
o And he was in possession of fake dollar checks in violation of
Art. 168 of the RPC.
Go Tek filed a motion to dismiss.
o The complaint was premature as he had a pending case in the
city fiscals office for violation of Art. 168.
o The board had no jurisdiction over the case because the board
may only deport aliens on the grounds expressly specified by law
citing an obiter in Qua Chee Gan.
The Board denied the motion ruling that a conviction is not
required before the State may deport an undesirable alien and that
the Board is only a fact finding body whose function is to make a
report and recommendation to the President.
Go Tek filed an action for prohibition with the CFI.
The CFI granted the petition by upholding the obiter in the
Qua Chee Gan case. It held that Sec. 37(3) of the Immigration Law
requires conviction of a crime involving moral turpitude and, thus,
the complaint was premature since mere possession of forged
dollar checks is not a ground for deportation under the Immigration
Law.
The Board appealed the decision to the SC.

ISSUES/HELD
Can the Deportation Board entertain a deportation proceeding
based on a ground which is not specified in section 37 of the
Immigration Law? YES.
Is prior conviction of the offense imputed to Go Tek necessary
to allow the board to continue its investigation? NO.

RATIONALE
Under existing law, the deportation of an undesirable alien
may be effected (1) by order of the President, after due
investigation, pursuant to section 69 of the Revised Administrative
Code and (2) by the Commissioner of Immigration upon
recommendation of the Board of Commissioners of the existence of
the ground for deportation, as charged against the alien, under Sec.
37 of the Immigration Law.
When deportation is effected by the President in the exercise
of his powers, it need not be under any ground specified in Sec. 37
of the Immigration Law; such a requirement is relevant only when
the deportation is effect by the Commissioner of Immigration.
o Sec. 69 of the RAC and E.O. No. 398, creating the Deportation
Board, do not specify the grounds for deportation
o There is no legal nor constitutional provision defining the
power to deport aliens because the intention of the law is to grant
38

You might also like